Motion in A Straight Line Subjective Questions
Motion in A Straight Line Subjective Questions
R
LiNE
SI
MOTION IN -A STRAIGHT
IT
~.1 MECHANICS Rest. An object is said to be at rest if it does not change
1. What is mechanics ? What are its various sub- its position w.r.t. its surroundings with the passage of time
branches? e.g., a book lying on a table.
Mechanics. Mechanics is the branch of physics that Motion. An object is said to be in motion if it changes its
deals with the conditions of rest or motion of the material objects position w.r.t. its surroundings with the passage of time e.g.,
H
a train moving- on rails. ' -,
around us. It is one of the oldest branches of physics.
Sub-branches of mechanics: Rest and motion are relative terms. A passenger
sitting in a moving train is at rest with respect to his
(i) Statics. It is the branch of mechanics that deals
fellow, passengers but he is in motion with respect to
with the study of objects at rest or in equilibrium, even
the objects outside the train. Thus an object may be at
when they are under the action of several forces. The
O
3.3 CONCEPT OF A POINT OBJECT an object moves along a plane. As shown in Fig. 3.2, the
4. What is meant by a point object ? Give suitable coordinates (xl' Yl) change to tx2, Y2) as the particle
moves from PI to P2 in a plane.
examples. ~
Point object. If the position of an object changes by
Examples of two dimensional motion :
distances much greater than its own size in a resonable (i) Motion of planets around the sun.
duration of time, then the object may be regarded as a point (ii) A car moving along a zig-zag path on a level road.
object. When a point object moves, its rotational and
Three dimensional motion. The motion of an object is
vibrational-motions may be ignored.
said to be three dimensional if all the three coordinates
Examples :- specifying its position change with time. Here an object
R
(i) Earth can be regarded as a point object for moves in space. As shown in Fig. 3.3, the coordinates
studying its motion around the sun. (xl' Yl' zl) change to (x2' Y2' z2) as the particle moves
(ii) A train under a journey of several hundred from Ii to P2in space.
kilometres can be regarded as a point object.
Z
SI
:,...;'DIMENSIONS, ..,'~
5. What do you mean by motion in one, two and
three dimensions ? Give examples of each type.
One dimensional motion. The motion of an object is
said to be one dimensional if only one of the three coordinates
specifying the position of the object changes with time. Here
the object moves along a straight line. This motion is
also called rectilinear or linear motion. As shown in
IT
Fig. 3.1, only the x-coordinate changes from Xl to x2
when the particle-moves from Ii to P~.along a straight r
line path. ' ..'
)
I' X2-----oj
r
.3.5 DISTANCE AND DISPLACEMENT
(1) Motion of a train along a straight trade i. t'
O
i f} I I; ld
pis~ance covered = AC + CB
, I
I--- XI ----I
I' X2 ~"--;"""'--I' ·1
.'..
Fig. 3.2 Two dtrnenslonal.motlon. Fig. 3.4
MOTION IN A STRAIGHT LINE 3:3
Distance is a scalar quantity because it has only (iii) Displacement is not dependent on the choice of the
magnitude and no direction. Distance covered is
always positive-or zero.
.
origin 0 of the position coordinates. As shown in Fig. 3.8, ,
R
displacement has both magnitude and direction, so it
is a vector quantity. I.i 'X2--...:...:::.------of
, --Jo
In Fig. 3.4, displacement = AB . ~Xl '1
--+1 ----tl~· -"'--_I -~""';"-I+-I-·L .1'J'
SI
, The SI unit of displacement is metre (m).
. t I! • J. \ {":.;. -. ~. ' r »: ~ I· . ·x2-'-----t·1 -,:
The CGS unit of displacement is centimetre (em). - i
j'.1 . . I .'
7. Mention some important characteristics of displace- Fig. 3.8, Shifting o~igin causes no change
ment. Give illustrations to support these characteristics. '. in displacement. , ,
Characteristics of Displacement: (iv) The actual distance Jravelled by an object in a given
(I) Displacement has the units of length. time interval is greater than or equal to the magnitude of the
(ii) The displacement of an object can be. posiiioe, displacement. The first situation is shown in Fig. 3.9 and
negative or zero. second one in Fig. 3.10. -
IT
When an object moves tow~rds right in tim~ ~'to t2,
its displacement is positive as shown in Fig. 3.5.
A
,.
~ Xl ----I
X2 -~':::="""I' , r
•A
Iii
o A B Fig. 3.9 Magnitude of displacement 6B (2 units)
I
H
<'Actual distance (O~ + AB) travelled (8 units).
I I
!
,.
Fig. 3.5 Positive,displacement. o
When an object moves.to the left in time tl to t2, its o A
t: . (,
displacement is negative as shown in Fig. 3.6.
O
1
2
1 1
, " ,
(v) The displacement of an object betuieen two points is
the unique path thai takes the body from its initial to final
position.
M
Fig. 3.6 Negative displacement. . (vi) The displacement of an object between two positions
When an object remains stationary, or it cioves first does not give any information regarding the sHape ~fthe
towards right and then an equal distance towards left, actual pathfollowed by the object betueen these two positions.
its displacement is zero as shown in Fig. 3.7. . (vii) The magnitude of the displacement of an object
, ! between two positions gives the shortest distance between
these positions.
(viii) Displacement is a vector quantity. Displacement of
3.6 ~ SPEED
8. Define the term speed. Is it a scalar or vector Average Speed In Different Situations
quantity ? Give its units and dimensions.
.., A body covering different distances with different
Speed. The rate of change of position of an object with speeds. Suppose a body covers distances sl' s2' h' ....
time in any direction is called its speed. It is equal to the
with speeds vI' v2' v3' .... respectively, then its average
. distance travelled by the object per unit time. speed will be'
Speed = Distance travelled Total distance travelled s
v = --=-----:----,------,---
Time taken av Total time taken
= sl + s2 + s3 + .
R
Speed has only magnitude and no direction, so it is
tl + t2 + 's + .
a scalar quantity. Also the distance travelled by an
object is either positive or zero, so the speed may be sl + s2 + s3 + ....
or vav = --'--""---"---
positive or zero but never negative.
The SI unit of speed is ms - 1. (
~+.2+~+ .... J
VI v2 . v3
SI
The CGS unit of speed is ems-I. Special case If sl = s2 =;0 s i.e., the body covers equal
The dimensional formula of speed is [tviOLIrI].
(iv) Instantaneous speed. The speed of an object at any Clearly, average speed is the arithmetic mean of the
particular instant of time or at a particular point of its path individual speeds.
is called the instantaneous speed of the object. Suppose an
object covers distance Sx in a small time interval 6.t
3.7 VELOCITY
around the instant t, then its average speed is tu I M.
The limiting value of this average speed when the time 10. Define the term velocity. Is it a scalar or vector
M
interval M approaches zero, gives the instantaneous quantity ? Give its units and dimensions.
speed at the instant t. Thus Velocity. The rate of change of position of an object with
Instantaneous speed, time in a given direction is called its velocity. It can also be
defined as the speed of an object in a given direction. It
v = lim tu = dx is equal to the displacement covered per unit time.
M~O Mdt · Displacement
V e Iocity = -..!....--.---
Here dx is the first order derivative of distance 'x Time
dt As velocity has both magnitude and direction, it is
with respect to time t.. a vector quantity. Velocity can be positive, zero or
The speedometer of ~r.automobile indicates its negative depending on the displacement is positive,
instantaneous speed at any instant. zero or negative.
MOTION IN A STRAIGHT LINE 3.5
R
intervals of time, however small these time intervals may be. 14. Obtain the formulae for the position of an. object
moving with uniform velocity v (i) at any time t in terms
(ii) Variable velocity. A body is said to be moving with
of its position at t = 0 and (ii) at any time t' in terms of
oariable velocity if either its speed changes or direction of
its position at another time t.
motion changes or both change with lime.
Kinematic formulae for uniform motion. Consider
(iii) Average Velocity. For an object moving with
SI
an object moving with uniform velocity v along the
variable velocity, average velocity is defined as the ratio of its
positive direction of x-axis. Suppose it occupies positions
total displacement to the total time interval in which that
XO' x and x' at timest =0, t and t' respectively, as shown
displacement occurs. in Fig. 3.11.
. . Total displacement
A verage ve Iocity = ----"---- xo x x'
. Total time
~~~~~--~I--------~I-----+I------'X
o 0 t t'
If Xl and x2 are the positions of an object at times t1
and t2, then the average veloeity from time t1 to t2 is Fig. 3.11 Uniform motion.
given by
IT
x -x !l.x Velocity of the object in the time interval 0 to tis
"v =.:L.....:! = -
av t2 - tl !l.t Displacement x - Xo
v= =--
(iv) Instantaneous velocity. The velocity of an object Time t-O
at a particular instant of time or at a particular point of its or x = Xo -i- vt . ...(1)
path is called its instantaneous velocity. It is equal to the
limiting value of the average velocity of the object in a This" equation gives position of" the particle at
H
small time interval taken around that instant, when the instant t.
time interval approaches zero. Thus Similarly, velocity of theobject in the time interval t
A-+ d-+ to t' is
it= Urn ~=~ x' -x
- M-+O M dt v=-- or x' = x + v (t' - t)
t' - t
O
~ (iii) Foruniform motion along a straight line in the Solution. (i) From A to. B. s _w d
same direction, the magnitude of the displace-
ment,is equal to the actual distance covered by
Distance covered ;=, l x 2 nr = t nr. <. :
.
' , .- . • '.
Displacement
(iv) The velocity is positive. if the object is moving
. =~,2 +;. =.J2r;
- ..
tbwards the right of the originand negative if
the object is moving towards the left of the origin. ; . ":
'v). For an object in uniform ,motio~'j~O force is Distance covered' = x 2 ttr = nr.
required to maintain its motion. _ ~ ". ~I.'\ : • ~
R
equal to the average velocity at all times because (iii) From A to D. -
veloqty remains C,O~stan~at each instant or at
Distance, co~ered = t x 2 ttr ~ ~ 1tT.
each point of the path ..
. ~ ..
16. What
*
I.
is• non-uniform
,
motion? Displacement = 1 AD 1 =),2 + ,:z :,:c.J2r ),
t . • It.· .• '! \
6bady ,.is.·said ·to .be in
SI
_i Non-uniform , motion,
-1' ,(iv)·I:'rom A ~0.4P~st~?ce,cov,ered b.~n;r- ,
non-uniform motion if its veloeity changes.with time. Here
either the speed of-the body orits direction of motion r.. As the- final'position coincides with the initial position,
or both change with time. For example, when a v~cle displacement covered = zero. '
starts moving from -rest, its velocity increases for EXAMPI.E 2. A_ca~ is ~oving along X-axis. As shown in
somenme. fhen its velocity may become constant for Fig. 3.13, it moves from 0 to P in 18 s and returns from P to
some time and finally slow down and come to rest Q in 6 p. Wha~ are the 'av~ragevelocity and average speed of
again. The velocity of the vehicle is different at iheccarin,goingjrom Ji)from 0 to,p"and(ii) from 0 to.P and
_different instants and so it lids norl!uniforln motion.' - back to Q ? [NCE~TJ
IT
40 80 120 160 200 240 280 320 360 400m
I I I I I .I I I I I .x
2 4 6 8 10 12 14 16 18 20 s
Q p
. . Time interval 18 + 6
EXAMJ>l.E 1. In Fig. 3.12, a + ~40m 10 -1 ..'
particle moves along a circular B :I' '·i ' = ms .
24-s; " {' .
path of 'radiusir: It; starts from ! I t 'll
one revolution. Also' 'find the KYtlMpI~B,3. A body traoels from A" t08 a.t,40'ms-1 arull
magnitude-of displacement in __ ••1••••• ··, from B to A,at 60·ms-1• Calculate the average speed and
each case. ' ,Fig. 3.12 . , average velocity. [Himachal 07]
MOTIONIN A STRAIGfff UN£ 3:1
. ., -~ -2 AS -- 2 AB ~ 1 or t2 =---
.:. Average' speed ;"-' - =---' =48 InS- . VI +,v2'
: I . '"t{+ tz AB/24
Total time. taken' •
As the body comes back to its iciti~lposition A, its ,x
R
X
+ tz = - + ---
, r
= tl
net displacement is ~ero. .' ~~ 2vo VI + V2
" ., . Displacement, ' '0 '
:. Average velocity = ' =---'=0. = x(VI + 'l!z +2vo)
, .' r - Time ·AB/24 '_.
2vo (VI + v2)
ExAlIfPI,E 4. On a 60 km track, a train travels thefirst 3:0}m
Average velocity
SI
with a uniform speed onO kmh-I. How fasi must ihe train Total distance x
travel 'the'next 30 kmso as to average' 40 'kmh-1 fo~ the
entire trip ?' . . ' _,
<"
,r Total time x(v,-f;v.:z+2vo)
2vo (VI + vz)
Solution. Here Sl'~ ~2.=s =39 km, PI =30'km h-r; = 2vo (VI + v2 )
vav=4Okmh-t, vz=?
• VI +~L+2vo
But v =_sI_+_s_<i
=_5_+_S_
aVt + t1. ~ + .!...
I
X PROBLEMS FOR PRACTICE'
vI 'vz
1. A 9'clist moving on a circular track of radius 100m
IT
.. 40 2 x 30 x Vz completes one revolution in 4 minutes. What is his
30 + Vz (i) average speed (ii) average. velocity in ore full
revolution? [ADS.(i) 50 1t metre/minute (ii) 0]
or 71
Vz =60 lon.-h • 2. A body travels the first half of the total distance
ExA.lt-fPLE 5. A
body covers one-third of its journey with with velocity q and the second half with velocity V)
L
I
speed 'u', next one-third with speed 'u' and the last one-third Calculate the average vel0c;ity. , . . 2q v2
H
with speed 'w'. Calculate the aoerage speed of the body Ans
C q + Vz
during the entire journey.
Solution. Let total distance = 3x. Then, 3. A car covers the first half of the distance between
two places at a speed of 40 kmh -1 and the second
.
Total time taken = -x + -x + -x = x (vw+uw+uv) half at 60 kmh - 1. What is the average speed of the
O
R
(I~..) . A verage ve I'
oaty = Displacement
. = 0. Different Types of Acceleration:
. . TIme
(i) Uniform acceleration. The acceleration of an object
2. Let total distance = 2x. Then, is said to be uniform acceleration if its velocity changes by
equal amounts in equal intervals of time, however small
Total-time taken = ~ + ~ = x (t} + V2) these tim~ intervals may be..
v2 t} v2
SI
t}
(ii) Variable acceleration. The acceleration of an object
2x 2iJ v
:. Average speed = = _1_2_ . is said to be variable acceleration if its pelocity changes by
x.(t}+v2) v1+V2 unequal amounts in equal intervals of time.
t} v2 . (iii) Average acceleration. For an object moving
3. v = 2t}v2 = 2 x 40 x 60 = 48 km h - 1• with variable velocity, the average acceleration is defined
av t}+v2 40+60 as the ratio of the total change in velocity of the object to the
total time interval taken. Suppose VI and v2 are the
4. Total distance covered . . velocities of an object at times tl and t2 respectively,
then its average acceleration is
IT
= 30 x 15 + 40 x 15 + 60 x 30 = 47.5 km
60 60 60 v2 -VI ·t:.v
a =---=-
Total time taken = 15 min + 15 min + 30 min av t -t !it
2 1
= 60min = Ih
(iv) Instantaneous acceleration. The acceleration of
Average speed = 47.5 = 47.5 km h-1. an object at a given instant of time' or at a given point of its
1 motion, is called its instantaneous acceleration. It is equal
H
5. Total distance covered = 1it+ s2 to the limiting value of the average acceleration of the
object in small time interval around that instant, when
Total time taken the time interval approaches zero. Thus
=t +t =.:i+~=1itv2+s2t} . t:.v dv
1,2 t} v2 t}v2 a= hm -=-
M-.O!it dt
O
Average velocity dx
,As V=-
_-,,1it_+_S--,,2~
= (1it+ S2)t} v2 dt
1itV2+S2t} 1itv2+S2t} 2
d (dX) dx
t} v2 a = dt dt = dt2
t t
SOx-+6Ox- Thus, acceleration is thefirst order derivative of velocity
M
6. Average speed = 2 2 = 55 kmh-1• and second order derivative of position with respect to time.
t
19. What are positive and negative accelerations? .
3~9 .•• ACCELERATION Give examples.
17. Define the term acceleration. Is it a scalar or a Positive acceleration. If the velocity of an object
vector quantity ? Give its units and dimensions. increases with time, its acceleration is positive. When a bus
leaves a bus-stop, its acceleration is positive.
Acceleration. The rate of change of velocity ~f an object
Negative acceleration. If the velocity of an object
with time is called its acceleration. It tells how fast the
decreases with time, its acceleration is negative. Negative
velocity of an object changes with time.
acceleration is also called retardation or deceleratiqn,
. Change in velocity When a bus slows down on approaching a bus-stop, its
A cce Ieration = --=----~
Time taken acceleration is negative.
MOTION IN A STRAIGHT LINE 3.9
(iii) Acceleration,
a = dv =..!! (18 + lOt) = 10 ms-2•
dt dt
EXAMJ>IJI 9. The displacement x of a particle varies with
time t as x = 4t2 -15 t + 25.
!ls ds
:2.,v= Lirn -=~dt Find the position, velocity and acceleration of the particle
M.-+QtJ.t
at t = O. When will the velocity of the particle become zero ?
.' . '. ~v dv d'4s Can we call the motion of the particle as one with uniform
4. a= Lim -=-=-
Afc-J~
.1,
at di de acceleration ?' 0
R
UNITSUSfD
All velocities are in ms-1 and a~~eleratibns in : . dx
Velocity, v = - =8t -15
ms-'-2 , ' di »
dv
EX.1MPLE z. The positfi:H'! of an b8ject moving along x-axis Acceleration, a= -
dt
=8
is given by x = Ii 'f- bt2, where a :::8.5 m b = 2.5 ms" 2 mid t
SI
is measured in :seconds, What is its velocity .at t ;, 0 sand At time t = 0, we have
t :::2 s ? What is theaveragevelocitybetween t = 2 s and t = 4 s ? x=25m, v=-15ms-1, d=8ms':'2•
[NCERT ; Himachal 08) Velocity will become zero, when 8t -15 =0
oSolution. Given x 0=a + bt2 15
or 0 t = - = 1.875 s.
Instanteneous veiocttj, 0
8
dx d 2 Yes,°the particle has a uniform acceleration because a
v=-=.-(a+bt )=0+bx2t=2bt
. dt dt 0 0 does not depend on time t.
At t =0, v =0. 10. The velocity of a particle is given by the
IT
EXAMJ>l"E
At t =1§, v == 2x 2.5x 2 =10 ms-1• equation, v = 2 t2 + 5 ems: 1. Find (i) the change in velocity
At t=2s,x=a+4b
of the particle during the time interval between t1 = 2 sand
t2 = 4 s (ii) the averag,eacceleration during the same interval
At t = 4s, x = a + 16b and (iii) the instantaneous acceleration at ti = 4 s.
Average velocity
·Solution. Given v =2t2 + 5 cm:s-1
=x2-x1 =(a+16b)-(a+4b)0=6b·
H
. t2 - t1 4-2 (i)When t1=2s, v1=2(2)2+5~13cms-1
v2=2(4)2+5=37cms-1
= 6 x 2.5 = 15.0 ms-1• When t2=4s,
Change in velocity
Example 8. The displacement (in metre) of a particle
moving along x-axis i~ given by x = 18t + St2. Calculate: 0
~ v2 - v1 =37 -13 = 2~ cms'".
(ii) Average acceleration,
O
dt dt dt dt
At t = 2 s, the instantaneous velocity At t = 4s, a =4 x4 =16 cms-2•
o Velocity, v = - =2t-6
t2 - t1 3-2 dt
3.10 ,-PHYSrCS-xt
I I I 'x
R
0 0 t
Solution. A~- a = (Iv =3t; ~ 2t + 2 vo V
,dt J
or
Integrating
dv = (3t2 -+ 2 t + 2) dt
both sides, we get
) . Fig._3.14
As shown in Fig.
Uniformly accelerated motion
3',1'4, let
f f -. "
.
' t
~.
SI
\
dv= (3t2 +2t+2)qt ,," i' ... ~ ,. ",.. 'Xo = position p(~th~ object at Instant t ,=0. _
t3 i2,..;- x='positiorroflh~oBjectatinstant'f
+ t2:+ 2 t + ,.C
• 0
r ~ "J.,; \ -
or -v = 3 - + 2 - +'2 t + C = t3
:: 3' ,2.'" ,- Vo =,velodry 'of the object at instant t =0
At t =0, v =2ms-1, therefore v = velocity of th~ object at 'insi.fnf' t' < I' I J
x - Xo = vav x t '.
acceler~tipn at f~J.~:_'" '(A~~. 36 ms-I, 14 ms" 2)
~ Initiahi-elodti -fI Final'velbaty i I
4. The distance trav~rs~'<iby a particle movin~ along a But vl/V = . 2
straight line is- given tby X,= 18().t .+, 50 t metre.
Find: = Vo + v
" 2
(i) the initial velocity of the particle
M
_vO+v _2vo_+at
(ii) tfie velocity at the end of 4s and x - Xo - -2- x t - 2 x t
(iii) the acceleration of the particle.
[.; v = Vo + at]
[Ans, (I) 180 ~s; 1 (ii) s80 ms-I (iil)IDO ms" 2]
r
- , t'
= vot +~ ai!
3.10 .:~ KtNEMAnC EQUATIONS FOR or X = Xo +- vot + '21 at 2 ~..(2)
UNIFORMLY ACCELERATED MOTION . Also, x - Xo = di~tanc~ travelled in time t = 5, say
,
20. Derive the following equations of motionfor
an object moving Wjth (X!'nstant acceleration alonq a ., 5 = vot + t at2 _..(3)
straight line : Equati0I1- (2) -is the required position-time relation
(i) v = Vo + at and equation (3) is the required distance-time relation.
", :1" < ••
R
t . dv
a=-
Multiplying equations (4) and (5), we get , dt
2 or dv = adt ...(1)
(v + va)(v - va) =- (x - xa) at
t When time = 0, velocity = u (say)
or v2 = 2 a (x = t, velocity = v
SI
- v~ - Xa) When time (say)
or V
2
= va2 + 2a (x - xa)
.'
...(6) Integrating equation (1) within the above limits of
time and velocity, we get
But x - xa = s, the distance travelled in time t, v t
therefore f dv = f a dt
...(7) u a
t
Equation (6) is the required velocity-position relation or [v]~ = a f dt=a[t]~
and equation (7) is the required velocity-distance relation. a
= a (1.;:,0)
IT
(iv) Distance (overed in nth second. As shown' in or- v - u
Fig. 3.15, the distance travelled 41 nth second can be
obtained by subtracting the distance travelled in first or v = u + at ...(2)
{n -1) seconds from the distance travelled in first n Second equation of motion. Velocity is defined as
seconds. ds
V=-
I
S"th
I dt
I I I
H
t=O t=n -1 t= n or ds = vdt =(u + at) dt ...(3)
I- 5"_1 'I When time = 0, distance travelled =0
,~
" I- s" 'I When time = t, distance travelled = s (say)
Fig. 3.15 Distance travelled in nth second. Integrating equation (3) within the above limits of
,.
time and distance, we get
O
o u
(iii) v2 - ,; = 2 as or v2 = ,; + 2as
v
J v dv
R
S
a
or aJ ds = (tv) <« =u+ "2(2n-l)
0 u
2. Equations of motion in Cartesian form,
or a [s]~ = [~2I (i) v(t) = v(O) + at
(ii) v(t')=v(t)+a(t'-t)
SI
v2 u2 (iii) x(t)=x(O)+v(O)t +!at2
or a[s-O]=---
2 2 (iv) x(t')=x(t)+v(t)(t'-t)+!a(t'-t,i
2 (v) [v(t')j2 - [v(t)2] = 2a [x(t') - x(t)].
or 2as = v - u2
UNITS USED
or v2 _ u2 =2as ...(6)
Displacement is in metre, velocity in ms" ',
Fourth equation of motion. By definition of acceleration in ms" 2 and time in second.
velocity,
ds 13. A jet plane starts from rest with an acce-
IT
ExAMPLE
V=-
dt leration of 3 ms- 2 and makes a run for 35 s before taking off.
or ds = vdt =(u + at) dt ...(7) What is the minimum length of the runway and what is the:
velocity of the jet at take off ?
When time = (n - 1) second, distance travelled = sn _ 1
Solution. Here u =0, a';"3 ms" 2, t =35s
(say)
Minimum length of the runway is given by
,When time = n second, distance travelled = sn
t at t x 3 x (35)2 = 1837.5 m.
H
2
(say) s = ut + =0 +
Integrating equation (7) within the above limits of Velocity of the jet at take off is
time and distance, we get v = u + at = 0 + 3'x 35 = 105 ms ".
r
S
J
n~
14. An electron travelling with a speed of
O
EXAMPLE
ds= (u+at)dt 3 1
5 x 10 ms- passes through an electric field with an
Sn -1 n-l acceleration ofl012 ms" 2. (i) How long will it take for the
n n electron to double its speed? (it) What will be the distance
or [s]:n
n -1
=u J dt + a J t dt covered by the electron in this time ?
n-l n-1' Solution. Here u = 5 x 103 ms -I,
M
EXAMPLE 15. A driver takes 0.20 s to apply the brakes after Solution. The situation is shown in Fig. 3.16.
he sees a need for it. This is called the reaction time of the
x=O SOm 7Sm 100m
driver. If he is driving car at a speed of 54 kmh" 1 and the I I I I
brakes cause a deceleration of 6.0 ms" 2, find the distance A
travelled by the car after he sees the need to put the brakes. t=0
R
2
u=15ms-l, v=0,a=-6.0ms-2
As v2 - u2 = 2 as
50 =0+ t x 1x ti
02 -152 = 2 x (- 6.0) s or ti = 100
or s = 225 =18.75 m .'. Time taken for first half run AB = tl = 10 s,
SI
12
For the motion from A to C :
Total distance travelled = 3.0+ 1&.75=21.75 m.
u=O, a=lms-2, s=75m
EXAlI'IPLE 16. On a foggy day two drivers spot each other
As s = ut + 1at 2
when they are just 80 metres apart. They are travelling at 2
72 km h- 1 and 60 km h- \ respectively. Both of them
applied brakes retarding their cars at the rate of 5 ms" 2.
75 = 0 + x 1 x t ti
Determine whether they avert collision or not. or t2 = .J155 = 12.2 s
Solution. For the first car: Time taken to run BC = 12.2-10= 2.2 s
IT
u=72 kmh-l =20ms-l, v=0,a=-5ms-2
Again for part AC, u =0, a =1 ms-2, t =12.2 s
As v2 - u2 = 2 as v = u + at =0 + 1 x 12.2 = 12.2 ms-1
.. 02 -202 =2 (-5)SI
For motion from C to D : This motion is uniform with
Distance covered by first car, sl = 40 m speed equal to 12.2 ms - 1.
For the second car:
H
.'. Time taken = 25 m = 2.04 s
u = 60 kmh - 1 = 60 x 5 12.2 s
18
Total time taken for 2nd half run BD
= -50 ms -1 ,v = 0,a = - 5 ms - 2
3 = 2.2 + 2.04 = 4.24 s.
As v2 -
2
u = 2 as: EXAiVIPLE 18. A motor car starts from rest and accelerates
O
R
As v2 _u2 =2as
s = vt _1 at2 = at . t _1 at2 = 1 at2
02 - 202 = 2 x a x 40 222
or a = - 400 = _ 5 ms - 2
80
or t= fI = ~2 ; 9 = 3 s
= 5 ms-:-2
SI
:. Retardation
s = vt = at. t = at2 = a. 2s =2s =2 x 9 = 18 m
J¥
v - u 0 -20 a
Time taken, t =-- =-- =4s
a -5 and v = at = a = ~2 as = ~2 x 2 x 9' = 6 ms -1.
Total time taken = 10 + 25 + 4 = 39 s.
EXA1HPI,E 21. A car acceleratesfrom rest at a constant rate
EXA.MPl,E19. An athlete runs a distance of 1500 m in the a for some time, after which it decelerates at a constant rate p
following manner. (i) Starting from rest, he accelerates to come to rest. If the total time elapsed is t second, then
himself uniformly at 2 ms" 2 till he covers a distance of calculate:
900 m. (ii) He, then runs the remaining distance of 600 m at
IT
(i) the maximum velocity attained by the car, and
the uniform speed developed. Calculate the time taken by the
athlete to cover the two parts of the distance covered. Also (ii) the total distance travelled by the car in terms of a, p
find the time, when he is at the centre of the track. and t.
Solution. The situation is shown in Fig. 3.17. Solution. (i) Let the car accelerate for time tl and v
t=0 t1 t2 be the maximum velocity so attained in time tl.
I I I As v = u + at
H
x=c 750m 900m 1500m
v =0 + atl
v
.1
As v2 - u2 = 2 as (t - tl)· Therefore,
vi - 02 = 2 x 2 x 900 o = v - p (t - tl)
or v2 =60 ms-l or t - tl =- v
... (2)
v-u 60-0 P
Time, t2 =--=--=30 s.
M
A.s s = ut + ~ at2 .' 750 =0 x tl +~x 2 x t~ (ii) Distance covered by the car in time tl is
1 2 1 v2 v2
or tl =~ = 27.4 s. Xl = 0 + - a tl = - a. 2 = - [using (1)]
. 2 2 a 2a
MOTION IN A STRAIGHT LINE 3.15
Distance covered by the car in time (t - tl) is Subtracting (3) from (2),
a y -z
X2 = V (t - tl) - 2"1 p (t - tl)
2
y-z=2"(2q-2r) or q-r=--
a
V l' v2 v2 Subtracting (1) from (3),
=v.---p.-=-
P 2 p2 2P [using (2)] a z-x
z-x=2"(2r-2p) or r-p=--
:. Total distance travelled by the car is a
x = xl + x2 = ;: + ;; = ~2 ( ± + ~)
Subtracting
a
(2) from (1),
x-y
R
x- y = - (2 P - 2 q) or p-q=--
a2p2t2 (a + P) 2 a
[using (3)]
2(a+p)2' ap Hence (q - r) x + (r - p) y + (p - q) z
2
apt
or X= . = (y-z)x +
(z-x)y (x-y)z
+ -'-----"--'---
2 (a + P) a a a
SI
EX.,11HPU' 22. A body covers. 12 m in 2nd second and 20 m = (xy - xz) + (yz - xy) + (xz - yz) = O.
in 4th second. How much distance will it cover in 4 seconds a
after the 5th second? [Chandigarh 03]
Lt_'WII; 24. Two buses A and B are at positions 50 m and
Solution. Here s2nd =12 m, S4th =20 m 100 III from the origin at time t = O. They start moving in the
a same direction simultaneously with uniform velocity of
10 1115-1 and 5 ms-l. Determine the time and position at
As Snth =u+2"(2n-l)
a which A overtakes B.
s2nd = U + 2" (2 x 2 -1) = 12
Solution. Here we use equation of motion for
IT
3 constant velocity in Cartesian form.
or u + - a = 12 ...(i)
2 - Given xl (0) = 50 m, x2 (0) = 100 m,
a
, v = 5-ms
1
Also s41h= u + 2" (2 x 4 -1) = 20 vI = 10 ms -1
2
(i) Velocity of the object at time t = 2 s is given by 7. Two trains-one travelling at 72 krnh -1 and other at
v(t) = v(O) + at 90 krnh - 1 are heading towards one another along a
straight level track. When they are 1.0 krn apart,
v(2)=3+4x2 =l1ms-1
both the drivers simultaneously see the other's
Position of the object at time t = 2 s is given by train and apply brakes which retard each train at
the rate of 1.0 ms - 2. Determine whether the trains
x(t) = x(O) + v(O) t + ~ at2 would collide or not. (Ans. No)
x(2) = 5 +3x 2 +1 x 4 x (2)2 = 19 m. 8. A bur~lar's car had a start with an acceleration of
2 2 ms - .A police vigilant party came after 5 seconds
, (ii) The position x(t)of the body when its velocity is and continued to chase the burglar's car with a uni-
R
5 ms-1 is given by form velocity of 20 ms - 1. Find the time in which the
2a [x(t) - x(O)] = v(t)2 - v(0)2 police van overtakes the burglar's car. (Ans.5 s)
9. A ball rolls down an inclined track 2 m long in 4 s.
2 x 4[x(t) - 5] = 52 _32
Find (i) acceleration -(ii) time taken to cover the
16
or x(t)-5 = 8 second metre of the track and (iii) speed of the ball
SI
at the bottom of the track.
or x( t) = 2 + 5 = 7 m. [Ans. (i) 0.25 ms" 2 (ii) 1.17 s (iii) 1 ms" 2]
10. A bus starts from rest with a constant acceleration
X PROBLEMS FOR PRACTICE
of 5 ms - 2. At the same time a car travelling with a
1. A race car accelerates on a straight road from rest to constant velocity of 50 ms - 1 overtakes and passes
a speed of 180 krnh - 1 in 25 s. Assuming uniform the bus. (i) Find at what distance will the bus
acceleration of the car throughout, find the distance overtake the car ? (ii) How fast will the bus be
covered in this time. [Delhi 02] travelling then? [Ans. (i) 1000 m (ii) 100 ms-1]
IT
(Ans. 625 m)
11. A body starting from rest accelerates uniformly at
2. A bullet travelling with a velocity of 16 ms-1 the rate of 10 ems - 2 and retards uniformly at the rate
penetrates a tree trunk and comes to rest in 0.4 m. of 20 ems - 2. Find the least time in which it can
Find the time taken during the retardation.
complete the journey of 5 km if the maximum
(Ans. 0.05 s) velocity attained by the body is 72 krnh -1.
3. A car moving along a straight highway with a (Ans. 400 s)
H
speed of 72 krnh -1 is brought to a stop within a 12. A body covers a distance of 20 m in the 7th second
distance of 100 m. What is the retardation of the car and 24 m in the 9th second. How much shall it
and how long does it take for the car to stop? cover in 15th s ? (Ans. 36 m)
[Delhi 95, OSC] 13. A body covers a distance of 4 m is 3rd second and
(Ans.2ms-2,.lOs) 12 m in 5th second. If the motion is uniformly
O
R
As v ==
u+ at
v2 - if = 2as
.. 20=
0+ G.Lx r, or t1 =2oos
or 3 =2x(-5)s
02_ (25)2 Also, if + 2 as
v2 =
.. 202 =
02 + 2 x 0.1 x 5 or 5 = 2000 m = 2 km
or = 6.94m
SI
5 (ii) For motion with uniform retardation:
Total distance travelled = 5.0 + 6.94 = 11.94 m. u = 20 ms - 1, V = 0,
8. Suppose the police van overtakes the burglar's car a=-20cms-2=-0.2ms-2, t=t2=? s=?
in time t second after it starts chasing. As V= u+ at
As 5 = ut + 1at2
2 Time, t 3 -- ~u -- 2000
20 -- 100 s
2 = 0 ~ ~ xa x 4
2
Total time = t1 + t2 + t3 = 200 + 100 + 100 = 400 s.
or a = 0.25 ms-2. 12. Here 2a· (2 x 7 -1)
O
57th =u+ = 20
(ii) Velocity v at the end of 1 m is given by 13
v2 - if as
= 2
or u+-a=20m ...(i)
2
or v2 02 = 2 x 0.25 x 1 = 0.5
-
a
13. s3rd = U + 2 (2 x 3 - 1) = 4
5
or u+-a=4
2 FORMULAE USED
a
S5th = U + 2 (2 x 5 - 1) = 12
1. For a freely falling body, the equations of motion
are' ..'
or u+ ~ a == 12 (i) v=-u-l; gt (ii).s=.ut + ~ gt2 (iii) ii -; =2gs
2
2. For a body falling freely under the action of
On solving,
gravity, g is taken positive.
U = - 6 ms -1, a = 4 ms - 2
3. For a body thrown vertic~lly upward, g is taken
Distance travelled in next 3 seconds . negative.
R
= s8 - s5 = [ - 6 x 8 + ~ x 4 x (8)2] 4. When a body is just dropped, U = 0
5. For a body thrown vertically up with initial
velocity u,' ," "':..' '.
-[ -6x5+~X4X(5)2].
(i) Maximum height reached, h = ;
= 80 - 20 = 60 m. 2g
SI
14. As v = u + at
(ii) Time of ascent = Time of descent = .!!.
:. In first case, 25 = U + 5a g
In second case, 34 = U + 8a (iii) Total time of flight = 2u
On solving, g
u=10ms-1, a=3ms-2 (iv) Velocity of fall at the point of projection = U
s= -h
earth from a small height is called free fall. The
acceleration with which a body falls is called Negative sign is taken
acceleration due to gravity and is denoted by g. because qisplacement is in the
opposite direction of initial
Near the surface of the earth, g = 9.8 ms" 2 velocity.
M
T
Equations.of motion for a freely falling body. For
As s = ut + ~ gt2
a freely falling body, the following equations of motion
hold good: :. - h = 19.6 x 6 + ~ x (- 9.8) x 62 h
from the ground. (i) How high will the ball rise? (ii) How KYA'HPLB 29. A balloon is ascending at the rate of9.8 ms-1
long will it be before the ball hits the ground? (iii) Trace the at a height of39.2 m above the ground when afood packet is
trajectory of this ball. [Central Schools 08, NCERT] dropped from the balloon. After how mucn time and with
Solution. (i) Here: u = + 20 ms ", g = -lOms-2 what velocitydoes it reachthe ground? Takeg = 9.8 ms" 2.
Solution. Initially the food packet shares the
At the highest point, v =0
upward velocity of the balloon, so
. Suppose the ball rises to the height h from the point
u=9.8ms-l, g=-9.8ms-2, s=-39.2m
of projection.
Here s is taken negative because it is in the opposite
As v2 - u2 =2gs
direction of initial velocity.
02 -202=2 x (-10)x h
t ,\it we get
R
2
or Using, s = ut + ,
h =+20 m.
(ii) Net displacement, s =-25m -39.2 =98 t - t x 9.8 t 2
t gt (t-4)(t+2)=0 or t=4s
s ~ •
or or -2 s
SI
\. ' . ......_ "''''' e.. ~.'
2
As s = ut + As time is never negative, so t = 4 s.
.. - 25 = 20 t + t x (-10) x t2 Velocity with which the food packet reaches the
or 5t2-20t-25=0 ground is
As s = ut + 1gt 2
Solution. For downward motion of the ball. Suppose
the ball falls from height hI and strikes the floor with
40 = 0 + 1x 9.8 x t 2
velocity vI' Then
R
Time taken,
u=v2, v=O, a=-g, s=h2
v-u 2-28
t=--=--=135 As v2
- u2
= 2 as
a -2
Distance, s = ut + 1at 2
=28 x 13 -1 x 2 x (13)2
.. 02-v~=2x(-g)h2
Average acceleration,
or v2=~2gh2
SI
=364 -169 =195 m
a = !':J.v = v2 - ( - VI )
Total time of parachutist in air !':J.t !':J.t
= 2.86 + 13 = 15.86 s
= v2 + VI = .j2iiI; + ..j2ih;
Height at which parachutist bails out !':J.t !':J.t
= 40 + 195 =235 m.
But hI = 4m, hz =3m, = 0.01 5,
!':J.t g = 9.8 ms" 2
EXA.lI1PLE 32. Two balls are thrown simultaneously, A
vertically upwards with a speed of20 ms-I from the ground, ~2 x 9.8 x 3 + ~2 x 9.8 x 4 2
.. a= = 1652 ms - .
and B vertically downwards from a height of 40 m with the om
IT
same speed and along the same line of motion. At what EXAlI-1PLE 34. A stone falls from a cliff and travels 24.5 m in
points do the two balls collide? Take g = 9.8 ms" 2. the last second before it reaches the ground at the foot of the
Solution. Suppose the two balls meet at a height of cliff. Find the height of the cliff.
x metre from the ground after time t s from the start. Solution. Here u =0, g = 9.8 ms-2
For upward motion of ball A :
Let nth second be the last second. Then
H
u = 20 ms -
s=ut+1gt2
I, g = - 9.8 ms - 2
u=20rns-1
Snth =U+ f (2 n -1)
or
For downward motion of a 4.9
ball B :
~ or n-=3 or t =3s
C
40 - x = 20 x t+ 1x 9.8 t 2
x
Distance, s = ut + 1gt 2
=0 + 1x 9.8 x 3 2
= 44.1 m
u=20rns-1 A
R
striking the ground? Take'g = 9.8 ms" 2. or
(Ans. 4s, 2s, 29.4 ms-I) or s = 3.2 m
4. A rocket is fired vertically from the ground with a .. The greatest height above the ground
resultant vertical acceleration of 10 ms" 2. The fuel = 85+ 3.2 = 88.2.
is finished in 1 minute and it continues to move up. (ii) For the downward motion:
SI
What is the maximum height reached- ? u=O, g=+10ms-2, 5=88.2m
(Ans. 36.4 km) 2 2
v = u + 2 gs = 0 + 2 x 10 x 88.2 = 1764
5. A balloon is ascending at the rate of 14 ms-I at a
.'. Velocity on reaching the ground, v = 42 ms-1.
height of 98 m above the ground when the food
packet is dropped from the balloon. After how (iii) Let t be the time taken to reach the maximum
much time and with what velocity does it reach the height.
ground? Take g = 9.8 ms" 2. For upward motion,
(Ans. 6.12s, 45.98 ms-I) u=8ms-I, v=O, g=-10ms-2
IT
6. A stone is dropped from a balloon rising upwards v-u 0-8
t = -- = -- = 0.8 s.
with a velocity of 16 ms -1. The stone reaches the g -10
ground in 4 s. Calculate the height of the balloon
4. Height covered in 1 min.
when the stone was dropped. (Ans. 14.4 m)
7. From the top of a tower 100 m in height a ball is ~ = ut + -t gt 2 =0+ -t x 10 x (60)2 = 18000m
dropped. and at the same time another ball is Velocity attained after 1 min,
H
projected vertically upwards from the ground with
v = u + at = 0 + 10 x 60 = 600 ms-I
velocity of 25 ms - 1. Find when and where the two
balls will meet. Take g = 9.8 ms" 2. [Delhi 05] After the fuel is finished, u = 600 ms -1, V = 0,
(Ans. 78.4 from top, 4 s) v2-u2=2gs
8. A body is dropped from rest at a height of 150 m, or 0-(600)2=2x(-9.8)xS2
O
remains constant at 50 m)
5 = ut + -t gt2
9. A body falling freely under gravity passes two
points 30 m apart in Is. Find from what point above - h = 16 x 4 - -t x 9.8 x 16 = -14.4 or h = 14.4 m.
the upper point it began to fall? Take g = 9.8 ms - 2.
8. For downward motion of the fir5t body:
(Ans. 32.1 m)
u=O, t=2s, g=10ms-2
10. Four balls are dropped from the top of a tower at
intervals of one-one second. The first ball reaches 5 = ut + 1 gt2 = 0 x 2 + 1 x 10 x 4 = 20 m
2 2
the ground after. 4 s of dropping. What are the Height from the ground, f; = 150 - 20 = 130 m
distances between first and second, second and For downward motion of the second body:
third, third and fourth balls at this instant? u = 0, t = 2 s, g = 10 ms -2.
(Ans. 34.3 m, 24.5 m, 14.7 m) 5= 0 x2 + -t x 10 x 4 = 20 m
3.22 PHYSICS-XI
Height from the ground, ~ = 100 - 20 = 80 m 24. Draw the position-time graph for an object in
Difference in heights = '1 - ~ = 130 - 80 = Sam uniform motion. Show that the slope of the position-
Similarly at t = 3 s, difference in heights = 50 m time graph gives the velocity of the object.
Hence the difference in heights does not depend on Position-time graph for uniform motion. An
time so long as both the bodies are in air. object in uniform motion covers equal distances in'
9. Suppose the body passes the upper point at t
equal intervals of time. So the position-time graph for an
second and lower point at (t + 1) s, then object in uniform motion along a straight line path is a
straight line inclined to the time-axis, as shown in Fig. 3.21.
52- ~ = t g (t + 1)2 - t gt2 = t g (2t + 1)
or 30m = t x9.8(2t + 1)
R
B
or t = 2.56 s ,
2 : X z -XI
.. ~ = t gt =t x 9.8 x (2.56)2 = 32.1 m.
e----.JR'
10. Here we need to find the distances travelled by the Iz-11
,,
:
SI
first ball in 4 s..by the second ball in 3 s,.by the third
ball in 2 s and by the fourth ball in 1 s. ,,
Xo A
,,
'1 = t g (4)2 = 8 x 9.8 = 78.4 m ;
~ = t g (3)2 = ~ x 9.8 = 44.1 m ;
graph for a stationary object is a straight line parallel to a parabola, as shown in Fig. 3.22.
the time-axis, as shown in Fig. 3.20.
,
t ,
t c
o
X
,
, dx
~ , ,
~§xo~--------------
'u; -r -'
'u; £ ,dl
,,
£
Slope e zz, ,,
o Time=e o 1
Time=-s
Fig. 3.20 Position-time graph for a stationary object. Fig. 3.22 Position-time graph for uniform acceleration.
MOTION IN A STRAIGHT LINE 3.23
: V2 -VI
= -dx = ve loci .
ocity at mstant t e I
dt -------IR
t2- tl :
Thus the slope of the position-time graph gives the I
I
instantaneous velocity of the object. Moreover, the I
,I.
slope of the x-t graph at time t = 0 gives the initial
velocity Vo of the object. o
R
3.14 VELOCITY-TIME GRAPHS Fig. 3.24 Velocity-time graph for uniform
acceleration.
26. Draw the velocity-time graph for an object in
uniform motion. Show that the area under the velocity- Slope of velocity-time graph AB
time graph gives the displacement of the object in the
SI
given time interval. = tan e = QR = v2 - VI
PR t2 - tl
Velocity-time graph for uniform motion. When an
object has uniform motion, it moves with uniform Change in velocity
velocity v in the same fixed direction. So the velocity- Time interval
time graph for uniform motion is a straight line parallel to = Acceleration (a)
the time-axis, as shown in Fig. 3.23- Hence the slope of the velocity-time graph gives the
acceleration of the object.
28. Show that the area under the velocity-time graph
IT
tAB of an object moving with constant acceleration in a
'g.c- v ~---T~--"""'- straight line in certain time interval is equal to the
distance covered by the object in that interval.
~
Distance covered as area under the velocity-time
D c graph. In Fig. 3.25, the straight line AB is the
o velocity-time graph of an object moving along a
H
Time~
straight line path with uniform acceleration a. Let its
velocities be Vo and v at times 0 and t respectively.
Fig. 3.23 Velocity-time graph for uniform motion.
= Area t \:
of rectangle ABCD .c- I
U I
= AD x DC = v (t2 - t1) ~
V
I
I
R
Fig. 3.26, its velocity-time graph is straight line. Here
OA = ED = u, OC = EB = v and OE = t = AD. ., A straight line graph has a single slope. So if the
displacement-time graph is a straight line, it represents
c a constant velocity. If the velocity-time graph is a
v ------------ B
I straight line, it represents a constant acceleration.
SI
I
t I
., A curved graph has multiple slopes. In Fig. 3.27, as
c- I
I the displacement-time graph bends upwards with the
passage of time, the value of e increases, slope
'0 I
o I
~ I
I (= tan e) of the curve increases, consequently the
I
I velocity increases and hence the motion is accelerated.
U -------------<D
A I
:E
o Tirne--.
IT
Fig. 3.26 Velocity-time graph for uniform acceleration.
= ut + t at x t or 5 = ut + t at 2
3.16 ANALYSING NATURE OF MOTION (vi) The distance covered by a body cannot decrease
FROM VARIOUS GRAPHS with the increase of time. So the distance-time
graph of the type shown in Fig. 3.34is not possible.
30. Discuss the nature of motion from the given
distance-time graphs.
J5bl'
Different Types of Distance-Time Graphs : I Not
t
(i) For a body at rest, the distance-time graph is a
straight line AB, as shown in Fig. 3.29. As the
slope of AB is zero, so speed of the body is zero.
R
Time~ Time~
A
t A B Fig. 3.34 Fig. 3.35
I
is
(vii) The distance-time graph shown in Fig. 3.35 is
not possible because it represents two different
SI
positions of the body at the same instant which
o Time ..• Time ..• is not possible.
31. Discuss the nature of the motion from the given
displacement-time graphs.
Fig. 3.29 Fig. 3.30
Different Types of Displacement-Time Graphs:
(ii) For a body moving with uniform speed, the (i) For a stationary body, the displacement-time
distance-time graph is a straight line inclined to graph is a straight line AB parallel to time-axis.
the time-axis, as shown in Fig. 3.30. As the graph The zero slope of line AB indicates zero velocity.
passes through 0, so distance travelled at t = 0 is
IT
also zero.
t
(iii) The distance-time graph in Fig. 3.31 represents
~ A B
accelerated (speeding up) motion, because the E
slope of the graph is increasing with time. ~
..!S
0...
is"'
H
A A
r t Fig. 3.36 0 Time ~
I
is
j
is'"
(ii) In Fig. 3.37, the displacement-time graph is a
straight line OA inclined to time-axis. It has a
single slope. So it represents a constant velocity
and hence zero acceleration.
O
o
A
Fig. 3.31 Fig. 3.32
r A
(iv) The distance-time graph in Fig. 3.32 represents
decelerated (speeding down) motion, because 1
M
t A
o Trme -e
R
Fig. 3.42 Fig. 3.43
(iii) In Fig. 3.44, the straight line v-t graph does not
pass through origin O. The body has an initial
velocity u (= OA) and then it moves with a
o Time=-e B uniform acceleration.
SI
Fig. 3.39 Fig. 3.40
A
(v) In Fig. 3.40, displacement is decreasing B
uniformly with time and becomes zero after a
certain time. Displacement-time graph has a
negative slope (8 > 90° and tan 8 < 0) and
represents a uniform negative velocity. It
indicates that the body is returning back to its
original position with a uniform velocity.
IT
(vi) In Fig. 3.41, the displacement of the body
becomes negative after time t and then increases Fig. 3.44 fig. 3.4~.
in magnitude with time. It indicates that the
body returning from position A, moves past the (iv) In Fig.. 3.45, greater .changes in velocity ·:are~
original position B andthen moves towards C . taking place. in equalintervals of time: SO-the. zkt
with a uniform velocity: graph bending upwards . represents. art .
H
A increasing.acceleration- '._
(v) In Fig. 3.46, 'smaller changes in velocity are
taking place in equal intervals of time. So the v-t
graph bending downwards represents a
decreasing acceleration.
O
A
A
Fig. 3.41-
32. Discuss the nature of the motion from the given
M
velocity-time graphs.
Different Types of Velocity-Time Graphs :
(i) For a body moving with a uniform velocity, the
v-t graph is a straight line parallel to the
time-axis as shown in Fig. 3.42. The zero slope
of line AB indicates zero acceleration. Fig. 3.46 Fig. 3.47
(ii) When a body starts from rest and moves with
uniform acceleration, its v-t graph is straight (vi) In Fig. 3.47, the body starts with an initial
line OA inclin~d to the time-axis and passing velocity u. The velocity decreases uniformly
through the orig.n 0, as shown in Fig. 3.43. with time, becoming zero after some time. As
Greater is the slope of the v-t graph, greater will 8 > 90°, the graph has a negative slope. The v-t
be the acceleration. graph represents uniform negative acceleration.
MOTION IN A STRAIGHT LINE 3.27
(vii) In Fig. 3.48, the v-t graph represents a body (ii) For a ball dropped on the ground from a certain
projected upwards with an initial velocity u. height, the speed-time graph is of the type
The velocity decreases with time (negative shown in Fig. 3.51. As the ball falls, its speed
uniform acceleration), becoming zero after increases. As the ball bounces back, its speed
certain time t. Then the velocity becomes decreases uniformly and becomes zero at the
negative and increases in magnitude, showing highest point.
body is returning to original position with
positive uniform acceleration.
R
~
'gol---~----
~
Fig. 3.51
SI
Fig. 3.48 34. Discuss the nature of motion from the given
(viii) The area between the velocity-time graph and acceleration-time graphs.
the time-axis gives the displacement. In Fig. 3.49, Different Types of Acceleration-Time Graphs :
the v-t graph represents variable acceleration. (i) For a body moving with constant acceleration,
the acceleration-time graph is a straight line AB
parallel to the time-axis, as shown in Fig. 3.52.
IT
t t A
.,~..
0
!U A B .,•..
~
0
~III
!U
III
u Qj
u
-e u
u
-<
Fig. 3.49
Displacement covered = Area 1 - Area 2 + Area 3
H
0 Time-+ Time-+
Distance covered = Area 1 + Area 2 + Area 3.
34. Discuss the nature of motion from the given Fig. 3.52 Fig. 3.53
speed-time graphs.
Different Types of Speed- Time Graphs: (ii) When the acceleration of a body increases uni-
formly with time, its a-t graph is a straight line OA
O
t
t §
I '.c
~Of--~r----+--~--
]
Time=-e
R
3. Distance travelled = Area between the (v-t) graph
and time-axis. ! 150 ,,
4. Change in velocity = Area between the (a-t) graph
. and time-axis. j<Il
100 - - -: R
,,
is ,
50
,
SI
Example .15. From the top of a tower, a ball is dropped to
'S
fall freely under gravity and at the same time; another ball is
2 3 4
thrown up with a velocity of 50 ms-1, Plot the position-time Time(h)~
graph for the motion of the two balls during the time interval
, t = 0 to t = 5s. Take g = 10 ms-2. Fig. 3.56
Solution. We can find the distances covered by the Solution. (i) B is ahead of A by the distance
two balls at different instants of time by using the OP = 100 km, when the motion starts.
formula, B ::::-=
QR
(II..) Spee d 0f 150 -100 ::::25 kmh-1 •
1 2
IT
S = ut+-gt PR 2-0
2
(iii) Since the two graphs intersect at point Q, so A
For the ball For the ball thrown up will catch B after 2 hours and at a distance of 150 km
' -1 from the origin.
dropped down u =50 ms ,
Time 0, 9 = + 10 ms-2
u :::: g=-10ms-2 (iv) Speed of A:::: QS:::: 150 -0 =75 krnh-1
OS 2-0
s =5t2 s ::::50t -5t2
H
:. Difference in speeds = 75 - 25 ::::50 kmh -1.
t = Is s=5m s=45m
Example 37. The speed-time graph of a particle moving
t =2s s=20m s = 'Jm
along a fixed direction is shown in figure. Find:
t=3s s=45m s = 105 m
(i) distance travelledby the particlebetween 0 sec to 10 sec
t =4s s=80m s = 120 m
O
125
100
75
50
t 25
S
~
O~~~--~~~--~
4 5 6
t (see)
>< 25 t (s) ~
Fig. 3.57
50
75 Solution. (i) Distance travelled by the particle
100 between 0 sec and 10 sec is
125
s ::::Area under speed-time graph
R
EXAil-IPU;; 40. A ball is thrown upward with an initial
Draw velocity-time graph of the body and find the total
velocity of100 ms-1. After how much time will it return?
distance travelled by the body. [Central Schools 04]
Draw velocity-time graph for the ball and find from the
Solution. The velocity-time graph is shown below. graph (i) the maximum height attained by the ball and
(ii) height of the ball after 15 s. Take g = 10 ms" 2.
u = 100 ms-1, g = -10 ms"
SI
Solution. Here 2
r At highest point, v = 0
As v=u+gt :. 0=100-10x t
:. Time taken to reach highest point,
t = 100 =10 s
2 ·34 5 6 7 8 10
!(see) ~
The ball will return to the ground at t = 20 s.
Fig. 3.58 Velocities of the ball at different instants of time
IT
will be as follows :
Distance travelled by the body
At t=O, v=100-10xO=100ms-1
= Area under v-t graph
1 At t=5s, v=100-10x5=50ms-1
= - (2 + 10) x 50 = 300 m.
.. 2 At t = 10 s, v = 100 -10 x 10 = 0
EXM1PLE 39. A train moves from one station to another in At t = 15 s, v = 100 -10 x 15 = - 50 ms-1
H
two hours' time. Its speed-time graph during the motion is At t = 20 s, v = 100 -10 x 20 = -100 ms-1
shown in Fig. 3.59. (i) Determine the maximum acceleration The velocity-time graph will be as shown in Fig. 3.60.
during the journey (ii) Also calculate the distance covered
during the time interval from 0.75 hour to 1 hour.
O
c t +50
50 ~ c
t
'00 Or-----~--~~----1-----~--
! 5 W
;> -so
t(s)~
-100
M
Fig. 3.60
(i) Maximum height attained by the ball
F 0
0.50 0.75 1.00 2.00 = Area of MOB
Time(h)~ = ~ x 10 s x 100 ms-1 = 500 m.
(ii) Height attained after 15 s
Fig. 3.59
= Area of MOB+ Area of LlBCD
Solution. (i) As the part BC of the speed-time graph
has maximum slope, so acceleration is maximum = 500 + ~ (15 -10)x (-50) =500 -125
during this interval. = 375 m.
3.30 PHYSICS-XI
40
Time ~
Fig. 3.63
2 3 4 5
Time (s) ~ AD v
R
Acceleration, a = slope of OA = - =-
OD OD
Fig. 3.61
OD=~
Solution. In time 0 to 2.5 s, acceleration of the a
vehicle,
Acceleration, p = slope of BC = BE =~
20 -0 2 EC EC
SI
a = Slope of OA=-- =8 ms"
2.5 -0
EC=~
In time 2.5 to 5 s, velocity of the vehicle is uniform. P
So acceleration = O. Now, total distance covered
In time 5 to 7 s, acceleration of the vehicle, = Area under the graph OABC
a = Slope of BC = --
0-20
= -10 ms
-2
or x = t [AB + OC] x AD
7-5
Hence the acceleration-time graph will be as shown = t [DE + 00 + DE + EC] x AD
IT
in Fig. 3.62. [.,' AB= DE 1
+ 10 a=8ms-2 = ~ [ 2 DE +~ + *] x v
t +5
a=O
or -=20E+-+-
2x
v p
v
a
v
OE=~[2: _~_*]
H
1 2 3 4 5 6 7
Time(s)~ or
-10
_._-
a=-10ms-2
Now, total time taken is given by
t= 00+ OE+ EC
O
2:_~_*]
Fig. 3.62
V
+-
EXAMPI.E 42. The velocity of a train increases at a constant = ~ +~ [ P
rate a from 0 to v and then remains constant for some time
interval and then finally decreases to zero at a constant rate
11 If the total distance covered by the particle be x, then show
or
t=;+¥[~+~l
M
6 -------~--------+---~
, " .1..10
,, "
"
4 , " '",
,,
R
2
________________ ..J
"
"I I g
r
O~~~--~~~~~~~~---
" ,
O~ __ ~ __ ~ __ ~ __ ~ __ ~, __ ~ __ ~" __ ~ __
·0
o
o 0.2 0.4 0.6 0.8 1.0 1.2 1.4 1.6 ~-10
t(h)~
-20
SI
Fig. 3.64
Fig. 3.65
(iv) How far did car A travel between the time it
7. As soon as a car just starts from rest in a certain
passed cars Band C ?
direction, a scooter moving with a uniform speed
(v) What is the relative velocity of car C with overtakes the car. Their velocity-time graphs are
respect to car A ? shown in Fig. 3.66. Calculate (i) the difference
(vi) What is the relative velocity of car B with
60
respect to car C ?
r A Car
[Ans. (i) C has the highest speed and A has the , 45 B
,
IT
lowest speed (ii) No (iii) 6 km from the origin g'" ,
(iv) 3 km (v) 7kmh-1 (vi) - 2 kmh-1] 'F Scooter
r30 ,, G
·0
2. An insect crawling up a wall crawls 5 em upwards 0 ,,
in the first minute but then slides 3 cm downwards ~ 15 ,
,
in the next minute. It again crawls up 5 em upwards ,
'C 'D
in the third minute but again slides 3 em
5 10 15 20 25
downwards in the fourth minute. How long will
H
Time(s)~
the insect take to reach a crevice in the wall at a
height of 24 em from its starting point? How does Fig. 3.66
the position-time graph of the insect look like? between the distances travelled by the car and the
(Ans. 21 min) scooter in 15 s (ii) the time when the car will catch
3. A driver of a car-travelling at 52 km h -1 applies the up the scooter and (iii) the distance of car and
O
brakes and decelerates uniformly. The car stops in scooter from the starting point at that instant.
5 seconds. Another driver going at 34 km h -1 applies [Ans. (i) 112.5 m (ii) 22.5 s (iii) 675 m]
his brakes slower and stops after 10 seconds. On the
8. The velocity-time graph of an object moving along
same graph paper, plot the speed versus time graph
a straight line is as shown below:
for two cars. Which of the two cars travelled farther
after the brakes were applied ?
M
A B
(Ans. Second car travelled farther)
4. A motor car, starting from rest, moves with uniform
acceleration and attains a velocity of 8 ms - 1 in 8 s. It
then moves with uniform velocity and finallybrought
to rest in 32 m under uniform retardation. The total
distance covered by the car is 464 m. Find (i) the
acceleration (ii) the retardation and (ii/) the total time
Time(s) ~
taken. [Ans. (i) 1 ms" 2 (ii) 1ms" 2 (iii) 66 s]
a
5. Starting from rest car accelerates uniformly with Fig. 3.67
3 ms - 2 for 5 s and then moves with uniform Calculate the distance covered by object between:
velocity. Draw the distance-time graph of the (i) t = a to t = 5 S (i/) t = a to t = 10 s.
motion of the car upto t = 7s. [Chandigarh 08] [Ans. (I) 80 m (ii) 130 m]
3.32 PHYSICS XI
X HINTS 60
A
1. (i) Slope of line C is highest and that of A is 50
lowest.
(ii) The lines do not meet at one point at any time.
(iii) A passes C at 0.6 h, at this time B is at 6 km
from the origin. "1:l
gj0- 20
14 - 6 -1
(v) VA = Slope of line A = 1.6 _ 0 = 5 km h Ul
10
R
14 - 2 -1
0
Vc = Slope of line C = 1.0 _ 0 = 12 km h 0 2 4 12
SI
-1
(vi) VB = Slope of line B = -- = 10 km h = Area of f1COD
1.4 - 0 . 52
EC 8
Sr = Area of toAOB
5. Distance covered by car in first 5 s is given by
= 1 OA x OB =1 x 52 km h-1 x 5s
5 = ut + 1 at 2 = 0 + 1 x 3 x t 2 = ~ t 2 1
= 1x 52 x 2. ms -1 x 5 s = 36.1 m.
2 18 Att = 5 s, v = u + at = 0 + 3 x 5 = 15 ms"
MOTION IN A STRAIGHT LINE 3.33
After t = 5 s, car covers a distance 15 m in each (iii) Distance travelled by the scooter in 22.5 s
second upto t = 7 s. = 30 ms-l x 22.5 s = 675 m.
Hence positions of the car at different instants of
Su :he car catches the scooter when both are at 675 m
time will be
I it'Ll the starting point.
S. (i) Distance covered between t = 0 to t = 5s
= Area OABDO= t(3+ 5) x20 = SO m.
Fig. 3.70 shows the distance-time (x-t) graph for the
(ii) Distance covered between t = 0 to t = 10 s
car. For accelerated motion (0 to 5 s), the graph is a
R
parabola (OA) and for uniform velocity (5 s to 7 s), = Area OABCO=t(3+ 10)x20=130 m.
the graph is straight line AB.
70
3.17 RELATIVE VELOCITY
B
35. Define relative velocity. Deduce an expression for
60 relative velocity of one object with respect to another in
SI
50 terms of their velocities relative to the earth.
t 40
Relative velocity. The relative velocity of an object 2
:§:>< with respect to object 1, when both are in motion, is the time
rate of change of position of object 2 with respect to that of
object 1.
Parabola Expression for relative velocity. As shown in Fig. 3.71,
1 consider the objects 1 and 2 moving along the same
O~~ __~ __~~ __~ __~ __~ _ direction with constant velocities vI and v2 (relative to
IT
123 4 5 6 7 the earth) respectively.
t(s)~
1 2
00-----_1 V1 0-0 ---- •• , V2
Fig. 3.70
7. (i) Distance travelled by the car in 15 s Subtracting (1) from (2), we find that
= Area of ~OAC x2 (t)xl (t) = X2 (0) - Xl (0) + (V2 - vI) t
-
3.~~ RELATIVE VELOCITY IN TERMS OF X2 (t) - Xl (t) is positive. The relative separation
POSITION-TIME GRAPHS x2 (t) - xl (t) first decreases till the two objects meet at
the position xl (t) = x2 (t). Then the separation
36. Draw and discuss the position-time graphs of two
objects moving along a straight line, when their relative x2 (t) - xl (t) becomes negative. The object 1 overtakes
the object 2 and the relative separation between them
velocity is (i) zero (ii) positive and (iii) negative.
again begins to increase.
Analysis of relative velocity in tenns of position-
time graphs. The relative velocity, v21 = v2 - vI may be
zero, positive or negative. t
§
Case 1. When the two objects move with same Position of
R
.0
velocity in the same direction. That is vI = v2 and ~ /. ~~e~~~ __
.. relative velocity, v2 - vI = 0
Then, x2 (t) - Xl (t) = X2 (0) - Xl (0)
SI
apart throughout their motion. Their position-time
graphs are parallel straight lines as shown in Fig. 3.72. Time of
Here the initial positions of the objects are xl (0) = 10 m meeting
and x2 (0) = 30 m. Velocities are vI = v2 = 5 ms - I. The
Time --.
two objects remain 20 m apart at all instants.
3.19 ~ DETERMINATION OF
IT
RELATIVE VELOCITY
37. Two bodies A and B are moving with velocities
v A and vB' making an angle e with each other.
Determine the relative velocity of A w.r.t. B. What will be
the relative velocity (i) when the two bodies move in
2 4 6 8 same direction and (ii) when the two bodies move in
Time (s)--.
H
opposite directions ?
Fig. 3.72 Position-time graphs for vI = v2 • Rule to determine relative velocity. The relative
Case 2. When v2 > vt or relative velocity (v2 - vt) is velocity of a body A with respect to another body B
when both are in motion can be obtained by adding to
positive. The relative separation x2 (t) - xl (t) increases
the velocity of A, a velocity equal and opposite to that
by the amount (v2 :: vI) after every second. So the
O
of B.Thus
position-time graphs gradually move apart, as shown
in Fig. 3.73. VAB=VA +(-vB)
Time --.
P'---:--~,.........----+..,..--p
Fig. 3.73 Position-time graphs for v2 > vI' VB
(a) (b)
Case 3. When V2 < vI or relative velocity (v2 - VI) is
negative. Initially the object 2 is ahead of object 1 and Fig.3.75 Determination of relative velocity.
MOTION IN A STRAIGHT LINE 3.35
with respect to B is given by the diagonal OR of the 1. Relative velocity of object A w.r.t. object B,
parallelogram OQRP'. --> --> -->
vAB = VA -VB
The magnitude of the relative velocity vAB is
2. Relative velocity of object B w.r.t. object A,
--> --> -->
VAB = ~v~ + v~ + 2 v A vB cos (180° - 9) VBA = VB -VA
R
=~v~ +v~-2vA vBcos9 .where 11A and vB are the velocities w.r. t. the ground
3. When the objects A and B move in the same
Suppose the relative velocity v AB makes angle p
direction,
with vA. Then --> --> -->
vAB = VA -VB
SI
VB sin (180° - 9)
tan p = -----"---~---'--- 4. When the object B moves in the opposite direction
vA + "s cos (180° - 9)
of A,
vB sin 9
VA =v« COS 9 UNITS USED
All velocities are in ms-1 or kmh-1.
or p = tan - 1 ( VA v! :~:s 9 J
EXAMPLE 43. A car A moving at 10 ms" 1 on a straight
IT
This gives the direction of the relative velocity v AB. road, is ahead of car B moving in the same direction at
6 ms-1. Find the velocity of A relative to B and vice versa.
Special cases : Solution. Here vA =10ms-1, vB=6ms-1
(i) When both the bodies are moving along Velocity of A relative to B,
parallel straight lines in the same direction. We have
9=0°.
vAB=vA -vB=1O-6=4ms-1
Positive velocity indicates that the driver of car B sees
H
VAB = ~v~ + v~ -2 VA VB cosO° car A moving ahead from him at the rate of 4 ms - 1.
Velocity of B relative to A
=~v~ +v~-2vA VB
vBA =vB-vA =6-10=-4ms-1
=~(VA -vB)2 =vA -vB· Negative velocity indicates that the driver of car A
O
R
(iii) Let velocity of monkey with respect to ground relative velocity of A w.r.t. B, -if (i) both cars are
=vM travelling eastwards and (ii) car A is travelling
:. Relative velocity of monkey with respect to train A eastwards and car B is travelling westwards.
[Ans. (i) 15 kmh - 1 eastwards
= vM -VA =-18 km h-1 =-5 ms-1
(ii) 105km h-I eastwards]
-5=15-5=10ms-t.
SI
or vM =vA
3. An open car is moving on a road with a speed of
EXAl.l1PLE 4.7' Two trains 120 m and 80 m in length are 100kmh -1. A man sitting in tIt car fires a bullet
running in opposite directions with velocities 42 kmh" 1 and from the gun in the opposite direction. If the speed
30 kmh - 1. In what time they will completely crosseachother ? of the bullet is 250 kmh -1 relative to the car, then
Solution. Relative velocity of one train w.r.t. the other find its (bullet's) speed with respect to an observer
on the ground. (Ans. 150 kmh-1)
= 42 -(-30) =72 kmh-1 =20 ms-1
Total distance to be travelled by each train to cross 4. A car A is moving with a speed of 60 kmh -1 and car
other train B is moving with a speed of 75 kmh -1, along
IT
parallel straight paths, starting from the same
= 120 + 80 = 200 m
point. What is the position of car A to.r.t. B after
Time taken by each train to cross other train 20 minutes? (Ans. 5 km behind)
200
=-=10s. 5. Two buses start simultaneously towards each other
20 from towns A and B which are 480 km apart. The-
EXAMPLE 46. The speed of a motor launch with respect to first bus takes 8 hours to travel from A to B while
still water = 7 ms - 1 and the speed vf stream is u = 3 me- ~. the second bus takes 12 hours to travel from B to A.
H
When the launch began travelling upstream, a float 'wds Determine when and where the buses will meet.
dropped from it. The launch travelled 4.2 km upstream, (Ans. 4.8 h, 288 km from A)
turned about and caught up with the float. How lo,;$.)s it 6. Two trains A and B,each of length 100m, are running
before the launch reaches the float ? r on parallel tracks. One overtakes the other in 20 s
Solution. For upstream motion of launch: and one crosses the other in 10 s. Calculate the
O
Relative velocity = 7 - 3 = 4 ms -
Distance moved = 4.2 km = 4200 m - 1. velocities of each train. (Ans. 15 ms" \ 5 ms-I)
7. A man swims in a river with and against water at
Time taken, t1 = 4200 = 1050 s the rate of 15 kmh - 1 and 5 kmh - 1. Find the many-
4 speed in still water and the speed of the river.
For downstream motion of launch : (Ans.10kmh-1,5kmh-I)
M
Distance moved downstream by float in 1050 s 8: A motorboat covers the distance between the two
= 3 x 1050 =3150 m spots on the river in 8 h and 12 h downstream and
Distance between rfloat and launch turned about upstream respectively. Find the time required by
the boat to cover this distance in still water.
= 4200 + 3150 =7350 m
(Ans. 9.6 h)
This distance is to be covered by launch with its
own velocity (7 ms" 1) because stream velocity is being 9. A car A is travelling on a straight level road with
shared by both. a speed of 60 kmh - 1. It is followed by another car
B which is moving with a speed of 70 kmh -1.
., Time taken, t =' 7350 = 1050 s
2 7 When the distance between them is 2.5 km, the
car B is given a deceleration of 20 kmh" 2. After
Total time taken, t = t1 + t2 =1050 + 1050 what distance and time will the car B catch up
=2100s=!~ with car A ? (Ans. 32.5 km, 0.5 h)
MOTION IN A STRAIGHT LINE 3.37
R
Speed of second bus = 480 = 40 kmh - 1 8
12 x
.Suppose the two buses meet after time t. Then u-v=- (for downstream)
12
60t + 40t = 480 On adding,
20 10
or t = 4.8 h 2u= - x or u=-x
SI
96 96
Distarcr from A = 60 x 4.8 = 288 km. .
Time required by boat in still water
6. Let ,vand v be the velocities of trains A and B x x
respectively. \
u
During overtaking, relative velocity of A w.r.t. B
9. Relative velocity of car B w.r.t. A
= u-v
=70-60=lOkmh-1
During crossing, relative velocity of A w.r.t B
.. For c r B,
=u+v
U= 10 kmh-1, 5 = 2.5 krn, a = - 20 kmh-1
IT
Total distance to be covered by A during over-
As s=ut+..!at2
taking or crossing 2
= 100 + 100 = 200 m 2.5 = 10t -..! x 20 x t 2 or t = 0.5 h
2
200 = 20 and 200 = 10
u-v u+v Actual distance travelled by car B during this time,
s = ut + ..! at 2 = 70 x 0.5 - ..! x 20 x (0.5)2
or u - v = 10 and u + v = 20
H
2 2
On solving, u = 15 ms"", v = 5 ms-1. = 35 - 2.5 = 32.5 km.
Problem 1. When does a. cyclist appear to be Problem 4. Can the speed of a body be negative?
stationary with respect to another moving cyclist? Solution. No, because the speed of an object is the
Solution. When both the cyclists are moving in the distance travelled per unit time and distance travelled is
same direction with the same velocity parallel to each never negative.
other. Problem 5. Can a body have a constant speed and
still have a varying velocity? [Himachal 06C]
M
Problem 8. Can a body have zero velocity and still be Problem 17. Two balls of different masses (one
accelerating? {Delhi OSC] lighter and other heavier) are thrown vertically upwards
Solution. Yes. A body thrown vertically upwards has with the same speed. Which one will pass through the
zero velocity at its highest point but has acceleration equal point of projection in their downward direction with the
to the acceleration due to gravity. greater speed ? [I. I. TJ
Problem 9. Can an object have an eastward velocity Solution. In case of motion under gravity, the speed
while experiencing a westward acceleration ? with which a body returns back is always equal to the
speed with which it is thrown up. Since expression for
Solution. Yes. A pendulum oscillating in east-west
final speed does not involve mass, both the balls will
direction will have eastward velocity and westward
acquire the same speed.
R
acceleration in half cycle of its oscillation.
Problem 18. Can the relative velocity of two bodies
Problem 10. Can the direction of velocity of an object
be greater than the absolute velocity of either body?
change, when acceleration is constant?
Solution. Yes. When two bodies move in opposite
Solution. Yes. For an object thrown vertically
directions, the relative velocity of each is greater than the
upwards, the direction of velocity changes during its rise
individual velocity of either body.
SI
and fall. But acceleration acts always downwards and
remains constant. Problem 19. A car travelling with a velocity of
50 km h -Ion a straight road is ahead of a motor-cycle
Problem 11. Is it possible for a body to be accelerated
without speeding up or slowing down ? If so, give an travelling with a speed of 75 kmh -1. How would the
example. [Himachal 01, 04] relative velocity be altered if motor cycle is ahead of
Solution. Yes. An object in uniform circular motion is car ?
accelerating but its speed neither decreases nor increases. Solution. The relative velocity will remain same as it
Problem 12. Under what condition is the average does not depend on the position of the two bodies.
velocity equal to the instantaneous velocity ? Problem 20. Even when rain is falling vertically
IT
Solution. When a body moves with a constant downwards, the front screen of a moving car gets wet
velocity, its average velocity over any time interval is while the back screen remains dry. Why ?
same as is instantaneous velocity. Solution. This is because the rain strikes the car in the
Problem 13. Why is the speed, in general, greater direction of relative velocity of rain with respect to car.
than the magnitude of the velocity ? Problem 21. Is it possible that the brakes of a car are
Solution. Because of the change in the direction of so perfect that the car stops instantaneously. If not,
H
motion, the length of the path traversed by a body is why?
generally greater than the magnitude of its displacement. Solution. No, it is not possible. In order to make
Sospeed is generally greater than the magnitude of velocity. velocity zero in an infinitesimally small interval of time,
Problem 14. Is the direction of acceleration same as the car needs an infinite declaration which is not possible.
the direction of velocity ? Problem 22. When an observer is standing on earth,
O
Solution. Not necessarily. If velocity increases, the trees and houses appear stationary to him. However,
acceleration acts in the direction of velocity and if velocity when he is sitting in a moving train, all these objects
decreases, then acceleration acts in the opposite direction appear to move in backward direction. Why ?
of velocity. Solution. For the stationary observer, the relative
Problem 15. Can we use the equations of kinematics velocity of trees and houses is zero. For the observer
to find the height attained by a body projected upwards sitting in the moving train, the relative velocity of houses
M
with any velocity ? and trees is negative. So these objects appear to move in
Solution. No. The equations of kinematics are backward direction.
applicable only so long as the acceleration is uniform. The Problem 23. The displacement of a body is given to
acceleration due to gravity is uniform only near the be proportional to the cube of time elapsed. What is the
surface of the earth. nature of the acceleration of the body?
Problem 16. Two balls of different masses (one Solution. Given: s oct3 or s = kt3
lighter and other heavier) are thrown vertically upward
with same initial speed. Which one will rise to the Velocity = ds = 3kt2
dt
greater height? [I. I. T.]
Solution. Both the balls will rise to the same heights. It Acceleration = dv = 6 kt i.e., acceleration oc t.
dt
is because, for a body moving with given initial velocity
and acceleration, the distance covered by the body does Clearly, the acceleration increases uniformly with
not depend on the mass of the body. time.
MOTION IN A STRAIGHT LINE 3.39
Problem 24. The average velocity of a partide is position-time graphs are parallel straight lines, equally
equal to its instantaneous velocity. What is the nature of inclined to the time-axis as shown in Fig. 3.76.
its displacement-time graph ?
Solution. When the average velocity is equal to the
instantaneous velocity, the particle has a uniform motion.
So its displacement-time graph is a straight line inclined t
c
to the time-axis. o
'.0
Problem 25. Can the position-time graph have a 'iii
negative slope? £
Solution. Yes,when the velocityof the objectisnegative.
Problem 26. What is the nature of the displacement-
R
time curve of a body moving with constant velocity ?
Solution. Parabola. Fig. 3.76
Problem 27. Can the direction of motion of a body
change if its velocity is changing at uniform rate? Problem 29. Is it possible to have a constant rate of
Solution. Yes.This can happen in a retarded motion. A change of velocity when velocity changes both in
magnitude and direction ? If yes, give one example.
SI
body thrown up moves under constant retardation and
from the highest point it begins to fall downwards. [Central Schools 08]
Problem 28. Draw position-time graphs for two Solution. Yes, in projectile motion, a body has
objects having zero relative velocity. uniform acceleration in the downward direction while its
Solution. As the relative velocity is zero, the two velocity changes both in magnitude and direction at every
bodies A and B have equal velocities. Hence their point of its trajectory.
outside the train. Rest and motion are relative terms. which gives the direction of motion of a body. When a
body is projected upward, both its direction of motion
Problem 2. If the displacement of a body is zero, is
and velocity are in upward direction but the acceleration
the distance covered by it necessarily zero ? Comment is in the downward direction.
with suitable illustration.
Problem 5. Two straight lines drawn on the same
Solution. No, it is not necessary that the distance displacement-time graph make angles 30° and 60° with
M
covered by a body is zero when its displacement is zero. . time-axis respectively, as shown in Fig. 3.77.Which line
Consider a particle moving along a circular track of radius represents greater velocity? What is the ratio of the two
r. After the particle completes one revolution, its velocities? [Delhi 05, 08]
displacement is zero while the distance covered is 21tr.
Problem 3. State in the following cases, whether the
motion is one, two or three dimensional :
(i) a kite flying on a windy day,
(ii) a speeding car on a long straight high way,
(iii) a carrom coin rebounding from the side of the
board, Fig. 3.77
3.40 PHYSICS-XI
R
Problem 6. If in case of a motion, displacement is 01-------
directly proportional to the square of the time elapsed, t .
what do you think about its acceleration i.e., constant or
variable? Explain why. [Chandigarh 02)
Solution. Given x oct2
SI
(c) (d)
or x = ct2,
where c is a constant Fig. 3.78
dx
Velocity, v= - = c x 2i
dt (a) If Xo = + ve and v = + ve, position-time graph will
dv be as shown in Fig. 3.78(a).
Acceleration, a = - = 2c = a constant
dt (b) If Xo = + ve and v = - ve, the position-time graph
Hence the object is moving with uniform acceleration. will be as shown in Fig. 3.78(b).
IT
Problem 7. Show that the average velocity of an object (c) If Xo = - ve and v = + ve, the. position-time graph
over an interval of time is either smaller than or equal to will be as shown in Fig. 3.78(c).
the average speed of the object over the same interval.
(d) If both Xo and v are negative, the position-time
Solution. We know that graph will be as shown in Fig. 3.78(d).
·
A verage ve Ioaty _D_is~p_Ia_c_e_m_e_n_t
= Problem 9. An object has uniformly accelerated
Time interval
motion. The object always slows down before the time,
H
= Total path length when its velocity becomes zero. Prove 'this statement
Average speed
Time interval graphically, when (a) both u and a are positive (b) u =- ve
As the magnitude of the displacement of an object can and a = + ve (c) u = + ve and a = -ve and (d) both u and a
be equal to or less than the total Iength of the path are negative.
traversed by the body-in given time, so average velocity of
O
Solution.
an object over an interval of time is either equal to or
smaller than the average speed of the object over the same (a) When both u and a are +ve : In such a case, the v-t
interval. graph will be as shown in Fig. 3.79(a}. At :the time
Problem 8. An object is in uniform motion along a corresponding to point A, the velocity becomes zero. It
straight line. What will be position-time graph for the can be seen that before this time, the velocity is negative
M
motion of the object if but its magnitude decreases with time till it becomes zero
at A.
(a) Xo = +ve, v = +ve
(b) Xo = +ve, v = -ve (b) When u is - ve and a is + ve : In this case, graph
(c) Xo = +ve, v = -ve
will be shown in Fig. 3.92(b}. At the time corresponding to
point A, the velocity becomes zero. It can be seen that
(d) both x 0 and v are negative?
before this time the velocity is - ve but its magnitude
The letters Xo and v represent position of the decreases with time till it becomes zero at A.
object at t =0 and uniform velocity of the object (c) When u + ve and a is - ve : In such a case, graph
respectively. between v and t will be as shown in Fig. 3.79(c). Again at
Solution.The position of the object moving with a A, velocity is zero. The velocity decreases before the time
uniform velocity v and time t is given by x = Xo + vt corresponding to point A.
MOTION IN A STRAIGHT LINE 3.41
,- ,-
-u
t'-t1=-z;.
a
A 0
(a) (b) Between times t ' and t2, the acceleration of the
object is
A 0 -v -0
... ... a = slope of v~ t graph BC = 2
+u ... t2 - t '
R
-u , v2
t2-t =--
a
Hence area under v-t graph between times tl and t2
-v -v
= ..! z;. (-~) -..! v (- V2)
(c) (d) 2 a 2 2 a
SI
2 2
v2 - Vi 2as
Fig. 3.79 =---=-=s
2a 2a
(d) When both u and a are - ve : In this case, v - t graph = Distance covered by the object between
will be as shown in Fig. 3.79(d). If we produce graph times tl and t2.
backwards, it meets the time-axis at point A. Before this Problem 11. Distinguish between distance and
time, velocity is +ve and decreases till it becomes zero at displacement. [Himachal 04, 06, 09C]
point A.
Solution.
Problem 10. The distance covered by an object
IT
between times t 1 and t 2 is given by the area under the Distance Displacement
1. Distance is the length of Displacement is the shor-
v - t graph between t 1 and t 2' Prove this statement for an
the actual path tra- test distance between
object moving with negative acceleration and having a versed by a body, irres- the initial and final
positive velocity at time t 1 and a negative velocity at pective of its motion. positions of body in a
time t2 given direction.
H
Solution. The velocity-time graph for the given 2. Distance between two Displacement between
points may be same or two given points is
motion of the object is as shown in Fig. 3.80. Let z;. be the
different for different always same.
velocity at time tl and - v2 at time t2• paths chosen.
3. It is a scalar quantity. It is a vector quantity.
O
Speed Velocity
Fig. 3.80
1. It is the distance It is the distance
Area under v-t graph between times tl and t2 travelled by a body per travelled by a body per
unit time in any unit time in a fixed
= Area AA'B + Area CC'B
direction. direction.
= 1A'B X AA' + 1BC' x CC' It is a scalar quantity.
2 2 2. It is a vector quantity.
= t (t' -. t1)(z;.) + t (t2 - t')( - v2) 3. Speed may be positive Velocity may be
R
Distance traversed in 3 s, or vA + vB =-
-t
s = ut + at = a + x 2 x (3l = 9 m.
2
-t tl
2x 2x
Problem 2. The acceleration experienced by a boat after Similarly, vB + Vc =- and Vc + vD =-
t2 t3
the engine is cut off, is given by dv = - kv3, where k is a V +v
dt Average velocity between A and D = A
SI
D
constant. If Va is the magnitude of the velocity at.cut off,find 2
the magnitude of the velocity at time t after the cut off. vA + "o
---'-'--2~"'- (tl + t2 + t ) = X + X + x 3
[CBSE 94]
r
3
v- dv =-k 6x 2x 2x 2x
or ----=---+-
vo 0
IT
tl + t2 + t3 tl t2 t3
3 1 1 1
or [ ~-22 = - k [t]~ or ----=---+-.
~l
vo tl + t2 + t3 tl t2 t3
Problem 4. In a car race, car A takes time t less than car
or - .!.[~ 2 - =- k [t - 0] B and passes thefinishing point with a velocity v more than
2 v va2 the velocity with which car B passes the point. Assuming
H
1 1 that the cars start from rest and travel with constant
or ---=2
2 2
kt
V Vo accelerations a1 and a2, show that v = t~ala2 .
1 _ 1 _1+2ktvo2 Solution. Let s be the distance covered by each car.
or 2 - 2 + 2 kt - 2 Let the times taken by the two cars to complete the
V Vo Vo journey be tl and t2, and their velocities at the finishing
O
1 1 1 3 2 2
- --+-=---- _1 2_1 2
Also s - "2 a1 tl -"2 a2 t2
tl t2 t3 tl + t2 + t3
2s 2s
Solution. As shown in Fig. 3.81, let the three successive
equal distances be represented by AB, BC and CD. Hence ~ = vI - v2 = tl t2 = 2s (t2 - t1) _ 2s
t2 - tl t2 - tl tl t2 (t2 - t1) tl t2
t=0 x x
I 4s2
tTt2
1 2
or v = t~a,. az .
Fig. 3.81
MOTION IN A STRAIGHT LINE 3.43
Problem 5. The driver of a train moving at a speed Vj Problem 7. A bullet fired into afixed target loses half of
observes another train at a distance d ahead of him on the its velocity after penetrating 3 em. How much further will it
same track moving in the same direction with a slower speed penetrate before coming to rest assuming that it faces
v2. He applies the brakes and gives his train constant constant resistance to motion ? [AIEEE 05]
. . (v -v )2 . Solution. In first case. If u is initial velocity, then
deceleration a. Show that if d > 1 2 , there ioill be no
2a v=u/2,s=3cm
2
2 As v2 - u2 = 2 as
collision and if d < (vI -v ) ,there will be collision.
2a (u /2)2 - u2 = 2 as or a = - u2 /8
Solution. There will be no collision if the driver of In second case: v = 0, a = - u2 /8
the train moving at a speed vI' reduces the speed of his
Initial velocity = u /2
R
train to v2 before the two trains meet i.e., the relative
velocity of the two trains must be reduced to zero. 02 - (u /2)2 = 2( - u2 /8) s
Initial relative velocity = vI - v2 or s= 1em
If s is the distance covered by first train before its Thus the bullet will penetrate a further distance of
relative velocity becomes zero, then 1 cm before coming to rest.
SI
v2 =u2 +2 as or 0 =(vI -v2)2 -2as Problem 8. A car, starting from rest, accelerates at the
(VI - v2) 2 rate f through a distance s, then continues at constant speed
or s = ---"--=--
2a for some time t and then decelerates at the rate f / 2 to come
2
to rest. If the total distance is 5 s, then prove that s = ft2. -t
For no collision, d > s or d>(vI-v2)
[AIEEE 05]
2a
Solution. For accelerated motion. u = 0, a = f, s = s
Problem 6. A juggler maintains four balls in motion,
making each in turn rise to a height of 20 m from his hand. As v2 - u2
= 2 as
IT
With what velocity does he project them and where will the vi _02 =2f s or VI =)2f s
other three balls be at the instant when the fourth one is just
leaving the hand? Take g = 10 ms-2. For uniform motion: u = VI =)2 f s, t = t
Solution. For upward motion of a ball : Distance travelled, s2 = ut = )2 f s t
v=O, a=-10ms-2, s =20m, u=?, t=? For decelerated motion: u = )2f s, a = - f / 2, v =0
As v2~u2=2as 0-u2=-2x10x20
As v2 - u2 =2as,
H
or u = 20 ms-1
02 -()2fd =2 x (- f /2)s3
Also, v = u + at 0=20-10t
Distance travelled, s3 =2s
or t=2s
Given s + s2 + s3 = 5s
So the ball returns tQthe hand of the juggler after 4 s.
f s t + 2s = 5s
O
or s + )2
To maintain proper distance, the balls must be
i = 1 s. or )2fst=2s
thrown up at an interval of
4 or s= t ft 2
•
When the fourth ball is in hand, the third ball has Problem 9. A car moving with a speed of 50 kmh-I can
travelled for 1 s, second for 2 s and first for 3 s.
be stopped by brakes after at/east 6 m. What will be the
-t at2
M
(i) For third ball, s = ut + minimum stopping distance, if the same car is moving at a
=20 x 1- -t x 10 x (1)2 = 15 m speed of100 kmh-I ? [AIEEE 03; Delhi 10]
Third ball. will be 15 m above the ground going Solution. In first case:
upward. u = 50 km h - 1 = 50 x ~ = 125 ms - 1. V = 0, s = 6 m
(ii) For second ball, s = 20 x 2 - -t x 10 (2)2 = 20 m 18 9 '
As v2 u2 = 2 as
J
-
Second ball will be 20 m above the ground and will
be at rest. 02 - ( 1~5 = 2a x 6
(iii) Forfirst ball, s = 20 x 3 - -t x 10 x (3)2 = 15 m
First ball will be 15 m above the ground going or a = _ 125 x 125 ""-16 ms" 2
81 x 2 x 6
downward.
3.44 PHYSICS-XI
R
18 18 g 9
instantaneous acceleration in terms of instantaneous
velocity. [AIEEE 05] Position of the ball after time T / 3,
Solution. Given: 2
t = ax + bx = h - -h = -,8h above the ground.
9 9
dt
-=2ax+ b Problem 12. Points P, Q and R are in a vertical line such
SI
dx
that PQ = QR A ball at P is allowed to fall freely. What is
Velocity, v = dx = (2ax + bf 1 the ratio of the times of descent through PQ and QR ?
. dt
[Central Schools 08]
. dv dv dx dv Solution. Let tl and t2 be the times of descent
Acceleration = - = - .- = v -
dt dx dt dx
through PQ and QR respectively.
= v~(2ax + bfl Let PQ = QR = h:
dx
= v (-1)(2ax + bf 2 .2a
Th en h -- "21gtl 2 an d 2h ="21 g(t + t ) 2
1 2
IT
t2
= -2a(2ax + bf3 =-2av3•
Problem 11. A ball is released from the top of a tower of
t = 1
(tl + t2)2
or
3.1. In which of theJoliowing examples of motion can the (iii) The spinning ball cannot be considered as point
O
body be considered approximately a point object: object because its size is quite appreciable as
(i) a railway carriage moving without jerks between compared to the distance through which it
two stations. turns on hitting the ground.
(il) a monkey sitting on the top of a man cycling smoothly (iv) The tumbling beaker slipping off the edge of a
on a circular track. table cannot be considered a point object because
(iii) a spinning cricket ball that turns sharply on hitting its size is not negligibly smaller than the height
M
o t~
Fig. 3.82
R
5.06 PM
,/
Ans. (Home)09~~1~0--1~1--~12~~~2~~3~-4~~5~~6--~7~
(a) As 0 P < OQ A lives closer to the school than B. AM Time-t PM
SI
the school earlier than B 3.4. A drunkard walking in a narrow lane takes 5 steps
forward and 3 steps backward, followed again by 5 steps forward
(c) Speed = Slope of x - t graph.
and 3 steps backward, and so on. Each step is 1 m long and
Slope of x - t graph > Slope of x - t graph requires 1 s. Plot the x-i graph of his motion. Determine
for B . for A. graphically and otherwise how long the drunkard takes tofall in
:. B walks faster than A a pit 13 m away from the start.
(d) Corresponding to the positions P and Q time t Ans. (a) Graphical method. Taking the starting point
is same on time-axis. as origin, the positions of the drunkard at various instants
:. A and B reach home at the same time. of time are given in the following table.
IT
(e) The x - t graphs for A and Bintersect each other
t (s) 0 5 8 13 16 21 24 29 32 37
only once. Since B starts from the school
afterwards, so B overtakes A on the road once. x (m) 0 5 2 7 4 9 6 11 8 13
3.3. A woman starts from her home at 9.00 A.M. walks with
'The position-time (x-t) graph for the motion of the
a speed of 5 kmh-1 on a straight road upto her office 2.5 km drunkard is shown in Fig. 3.84. As is obvious from graph
away, stays at the office upto 5 P.M. and returns home by an that the drunkard would take 37 s to fall in a pit 13 m
H
auto with a speed of 25 kmh - 1. Choose suitable scales and plot away from the starting point.
the x-t graph of her motion.
Ans. For the journey from home to office:
The time at which woman leaves for her office is 9 AM. 14 X= 13 m
12
As she travels with a speed of 5 kmh -1 and the
O
t=37s
Between 9.30 AM. to 5.00 P.M., she stays in her office
i.e. at a distance of 2.5 km from her home. 8 10 12 14 16 18 20 22 24 26 28 30 32 34 36 38
t (s) ~
For the return journey from office to home:
Fig. 3.84
The time at which she leaves her office =5 P.M.
(b) Analytical method. In each forward motion of 5
Now she travels a distance of 2.5 km with a speed of
25 kmh - 1, hence the time taken steps and backward motion of 3 steps, net distance
covered = 5 - 3 = 2 m and time taken = 5 + 3 = 8 s.
2.5km 1 .
t' = 1 = - h = 6 minutes :. Time required to cover a distance of 8 m
25kmh 10
= ~ x8= 32s
The time at which she reaches her home = 5.06 P.M. 2
3.46 PHYSICS-XI
Remaining distance of the pit = 13 - 8 = 5 m 3.8. On a two-lane road, car A is travelling with a speed of
36kmh-1. Two cars Band C approach car A in opposite
In next 5 s, as he moves 5 steps forward, he falls into
directions with a speed of 54 kmh-1 each. At a certain instant,
the pit.
when the distance AB is equal to AC, both being 1 km, B decides
:. Total time taken = 32 + 5 = 37 s. to overtake A before C does. What minimum acceleration of car
B is required to avoid an accident?
3.5. A jet airplane travelling at the speed of 500 kmh-1
ejects its products of combustion at the speed of 1500 kmh-1 Ans. At the instant when B decides to overtake A, the
relative to the jet plane. What is the speed of the latter with speeds of three cars are
respect to an observer on the ground?
vA = 36kmh-1 =36 x~ =+ 10ms-1
Ans. Here speed of jet airplane, 18
'1. = 500 kmh-1 vB = + 54kmh-1 =+ 15 ms-1
R
= + 54 x~
Let v2 be the speed of products w.r.t. the ground. 18
Suppose the direction of motion of the jet plane is positive.- Vc = - 54 kmh - 1 = - 15 ms - 1
Then the relative velocity of products w.r.t. jet plane is Relative velocity of C w.r.t. A,
v2 - '1. = -1500 vCA = Vc - VA = - 15 - 10 = - 25 ms - 1
or v2 = '1. - 1500 = 500 - 1500 = - 1000 kmh-1
SI
:. Time that C requires to just cross A
Negative sign shows that the direction ofproducts of 1km 1000 m
combustion is opposite to that of the jet plane. =--= 1 =40s
vCA 25 ms"
:. Speed of products of combustion w.r.t. ground
= 1000 kmh-1.
= 54 krnh-I = 36 krnh-I = - 54 krnh-I
r- -I
VB VA Vc
B just brushes past the driver of A, what was the original 3.9. Two towns A and B are connected by a regular bus
distance between them ? service with a bus leaving in either direction every T min. A
Ans. Let x be the distance between the driver of train man cycling with a speed of 20 kmh-1 in the direction A to B
A and the guard of train B. Initially, both trains are mOvin notices that a bus goes past him every 18 min in the direction of
in the same direction with the same speed of 72 km h - . f his motion, and every 6 min in the opposite direction. What is
So relative velocity of B w.r.t. A = vB - vA = O. Hence the the period T of the bus service and with what speed (assumed
train B needs to cover a distance with constant) do the buses ply on the road?
a=lms-2, t=50s, u=O Ans. Let speed of each bus = v kmh-1
Buses plying in this direction go past the cyclist after (iii) When the highest point is chosen as the location
every 18 min. for x = 0 and t = 0 and vertically downward direction to be
:. Distance covered the positive direction of X-axis:
18 During upward motion. Position is positive, velocity is
= ( v - 20) 60 km
negative and acceleration is positive.
Since a bus leaves the town after every T min, so the During downward motion. Position is positive, velocity
above distance covered is positive and acceleration is positive.
T (iv) For upward motion.
=vx-km
60 u=-29.4ms-l, g=+9.8ms-2, v=O
R
18 T If s is the height to which the ball rises, then
.. (v-20)-=vx- ... (i)
60 60 v2-if=2as
For buses going from town B to A : or 02 - (- 29.4)2 = 2 x 9.8 x s
Relative speed of bus in the direction opposite to the s = - (29.4)2 = _ 44.1 m
motion of the man or
2 x 9.8
SI
= (v + 20) kmh - 1
Negative sign shows that the distance is covered in
Buses going in this direction go past the cyclist after
upward direction.
every 6 min, therefore
6 T If the ball reaches the highest point in time i, then
(v+20)-=vx- ... (ii)
60 60 v = u + at
(ii) What are the velocity and acceleration of the ball at (a) True. When a body begins to fall freely under
the highest point of its motion ? gravity, its speed is zero but it has non-zero
(iii) Choose the x = 0 and t = 0 to be the location and acceleration of 9.8 rns" 2.
time of the ball at its highest point, vertically (b) False. Speed is the magnitude of velocity and
downward direction to be the positive direction of the magnitude of non-zero velocity cannot be
M
3.12. A ball is dropped from a height of 90 m on a floor. At Again the speed of the ball increases linearly with
each collision with the floor, the ball loses one-tenth of its speed. time t from 0 to 37.8 ms-1 (initial speed of the previous
Plot the speed-time graph of its motion between t = 0 to 12 s. upward motion) in the next time-interval of 3.9 s. Total
Ans. (i) Time taken by the ball to fall through a height time taken from the start = 4.3+ 3.9+ 3.9= 12.1 s. This part
of 90 m is obtained as follows: of motion has been shown by straight line CD.
x = v (0) t + -t gt 2 Here, we have assumed a negligible time of collision
between the ball and the floor.
90= 0+ -t x9.8t 2
3.13. Explain clearly, with examples, the distinction between :
t = ~ 2 x 90 = 30 s :::4.3 s (a) magnitude of displacement (sometimes called distance)
or
9.8 7 over an interval of time, and the total length of path
R
Now v( t) = v( 0) + gt covered by a particle over the same interval;
(b) magnitude of average velocity over an interval of
v (4.3) = 0 + 9.8 x 30 = 42 ms-1
7 time, and the average speed over the same interval.
From time t = 0 to t = 4.3s, [Average speed of a particle over an interval of time
is defined as the total path length divided by the
v (t ) = gt = 9.8 t or v (t ) oc t
SI
time interval}.
In this duration speed increases linearly with time t
(c) Show in both (a) and (b) that the second quantity is
from 0 to 42 ms -1 during the downward motion of the
either greater than or equal to the first. When is the
ball and this speed-time variation has been shown by
equality sign true ru» simplicity, consider
straight line OA in Fig. 3.86.
one-dimensional motion only].
Solution.
~':.'!.2_n.:!(~
_A
40 v = 37.8 ms -1 First collision (a) Suppose a body moves from point A and to
35 -------B --------------- Second point B along a straight path and then returns
collision
IT
back to the point A along the same path.
t 30 E
As the body returns back to its initial position
'", 25 A, so magnitude of displacement = 0.
Ei
'l>' 20 Distance covered
15 = Total length of the path covered
10 =AB+BA=AB+AB=2AR
H
(b) In the above example, suppose the body takes
time t to complete the whole journey. Then
Magnitude of average velocity
Ans. Case (i) : 0 to 30 min 3.15. The instantaneous speed is always equal to the
Speed = 5 kmh-1 magnitude of instantaneous velocity. Why ?
Distance covered in 30 min Ans. Instantaneous speed,
. L\x
= 5 kmh - 1
30 h = 2.5 km
x V= L1m-
M-40tlt
60
Displacement covered = 2.5 km Instantaneous velocity,
-4
(a) Average velocity L'
-4
V= 1m--~ x
"'I -4 a tlt
= Displacement = 2.5 km = 5 kmh-1.
R
Time 30/60 h For an arbitrary small interval of time (tlt), the
magnitude of displacement t ~ t is equal to the length of
(b) Average speed
the path L\x. So instantaneous speed is always equal to the
Distance 2.5 km = 5 kmh-1. magnitude of instantaneous velocity.
Time 30/60 h 3.16. Look at the graphs (a) to (d) [Fig. 3.87] carefully
SI
Case (ii) : 0 to 50 min and state, with reasons, which of these cannot possibly
represent one-dimensional motion of a particle.
Displacement covered in first 30 min in going to
market x v
= 5 kmh - 1 30 h = 2.5 km
x
60
Displacement covered in next 20 min in coming back
to home
= 7.5 kmh-1 20 h = 2.5 km
x
IT
60
(a) (b)
Net displacement = 2.5 - 2.5 = 0
Total distance covered = 2.5 + 2.5 = 5 km Speed
Total path
length
(a) Average velocity
= Net displacement = 0 = o.
Time taken 50 / 60 h
H
(b) Average speed
Total distance 5km =6kmh-1.
(c) (d)
Time taken 50/60h
Case (iii) : 0 to 40 min Fig. 3.87
O
3.17. Fig. 3.88 shows the x-t plot of one-dimensional motion 3.19. Suggest a suitable physical situation for each of the
of a particle. Is it correct to say from the graph that the particle following graphs [Fig. 3.89] :
moves in a straight line for t < 0 and on a parabolic path for x
t > 0 ? If not, suggest a suitable physical context for this graph.
A R
5
t... p Q B
T u
1<0
R
(a)
t~
v a
Fig. 3.88
SI
Ans. No, it is wrong to say that the particle moves in a
straight line for t < 0 and on a parabolic path for t > 0,
because a position-time (x - t) graph does not represent (b) (e)
the trajectory of a moving particle.
Fig. 3.89
This graph can represent the motion of a freely falling
particle dropped from a tower when we take its initial Ans.
position as x = 0, at t = O. (a) A ball lies at rest on a smooth floor, as indicated by
3.18. A police van moving on a highway with a speed of 30 straight line PQ of the x-t graph. It is kicked
r:
IT
kmh-1 fires a bullet at a thiefs car speeding away in the same towards a wall. Slope of line QR gives speed of
direction with a of 192 kmh-1 .If the muzzle speed of the kicking. The ball rebounds from the wall with a
bullet is 150 ms" .unth what speed does the bullet hit the thiefs reduced speed (given by the slope of line RS). At S,
car? the sign of position coordinate changes sign, it
indicates that the ball moves to the opposite wall
Solution. Speed of police van,
which stops it. The line TU indicates the final rest
-125 -1 position of the ball.
v =30kmh =- ms
H
p 3 (b) The velocity-time graph represents the motion of a
Speed of bullet, ball thrown up with some initial velocity, it hits
Vb = 150 ms-1 the ground and gets rebounded with a reduced
speed. It goes on hitting the ground and after each
Speed of the police van is shared by the bullet.
hit its speed decreases unit it becomes zero and the
:. Relative speed 01 bullet w.r.t. ground
O
3
Speed of thief's car, position, velocity and acceleration variables of the particle at
t = 0.3 s, 1.2s, - 1.2s.
v = 192 kmh-1 = 160 ms-1
t 3
x
Relative speed of bullet w.r.t. thief's car
= (vb + vp)-vt
475 160
---
3 3
= 105 ms-I.
Ans. The acceleration of a particle executing S.H.M. is constant direction of motion, give the signs of v and a in the
given by three intervals. (d) What are the accelerations at the points A, B,
a = - (f}X C and D ?
where co (angular frequency) is a constant.
At time t = 0.3 s D
r
I
As is obvious from the graph, x < 0
As slope of x-t graph is negative, so v < 0
As a = - co2x, so a > O.
R
At time t =1.2 s
-
As is obvious from the graph, x > 0 1 2 3 t~
SI
Ans.
At time t=-1.2 s
(a) As the change in speed is greatest in interval 2,
As is obvious from the graph, x < 0 so magnitude of average acceleration is
As slope of x - t graph is positive, so v > 0 greatest in interval 2.
As a = - co2x, so a > O. (b) Obviously from the graph, average speed is
greatest in interval 3.
3.21. Figure 3.91 gives the x-i plot of a particle in one-
dimensional motion. Three different equal intervals of time are (c) vis positive in all three intervals. a is positive in
the intervals 1 and 3 as speed is increasing in
shown. In which interval is the average speed greatest, and in
these intervals. But a is negative in interval 2 as
IT
which is it the least? Give the sign of average velocity for each
speed is decreasing in this interval.
interval.
(d) At the points A, B, C and D, the u-t graph is
parallel to the time-axis or has a zero slope, so
a = o at all these points.
3.23.A three-wheeler starts from rest, accelerates uniformly
with 1 ms 2 on.a straight road for 10 s, and then moves with
H
uniform velocity. Plot the distance covered by the vehicle during
t~ the nth second (t} = 1, 2, 3... ) versus n. What do you expect this
plot to be during accelerated motion : a straight line or a
parabola?
Ans. Distance travelled in nth second,
O
a
Sll III = U + - (2n - 1)
2
Fig. 3.91 u 0, a = 1 ms - 2
As =
= 1(2n-l) m
2
As the slope of x-i is greatest in interval 3 and least in
Thus the distances travelled by the three-wheeler at
interval 1, so the average speed is greatest in interval 3
the end of each second are given by
and least in interval 1.
As the slope of x-i is positive in intervals 1 and 2 and n(s) 1 2 3 4 5 6 7 8 9 10
negative in interval 3, so average velocity is positive in sn th (m) 0.5 1.5 2.5 3.5 4.5 5.5 6.5 7.5 8.5 9.5
intervals 1 and 2 and negative in interval 3.
Now, velocity of the three-wheeler at the end of 10th s
3.22. Figure 3.92 gives a speed-time graph of a particle in
is given by
motion along a constant direction. Three equal intervals of time
are shown. (a) In which interval is the average acceleration v = u + at = 0 + 1 x 10 = 10 ms - 1.
greatest in magnitude ? (b) In which interval is the average Upto n = 10 s, the motion is accelerated and the graph
speed greatest ? (c) Choosing the positive direction as the between Sllth and n is a straight line AB inclined to
3.52 PHYSICS-XI
R
::-.~ I
-----------.~~~ I
I
Fig. 3.94
(ii) Speed of the child running opposite to the
direction of motion of the belt
=(9-4)kmh-I =5 kmh-1•
SI
(iii) Speed of the child w.r.t. either parent
345 10 11 12 13 = 9 kmh - 1 = 2.5 ms - 1
n(s)~
Distance to be covered = 50 m
Fig. 3.93 T·lffie t ak en = -50 = 20 s.
2.5
3.24. A boy standing on a stationary lift (open from above) If the motion is viewed by one of the parents, answers
throws a ball upwards with the maximum initial speed he can, to (i) and (ii) are altered but answer to (iii) remains
equal to 49 ms-1. (i) How much time does the ball take to return unchanged.
to his hands? (iz) If the lift starts moving up with a uniform speed
3.26. Two stones are thrown up simultaneously from the
IT
of 5 ms - 1, and the boY again throws the ball up with the maximum
edge of a cliff 200 m high with initial speeds of 15 ms-I and
speed he can, how long does the ball take to return to his hands?
30 ms-I. Verify that the following graph correctly represents
Ans. (i) When the lift is stationary: For upward motion the time variation of the relative position of the second stone
of the ball, we have with respect to the first. Neglect air resistance and assume that
u=49ms-I, g=-9.8ms-2, v=O, t=? the stones do not rebound after hitting the ground. Take
As v= u+ at g = 10 ms - 2. Give the equations for the linear and curved parts
H
49 of the plot.
0=49-9.8t or t =- =5s.
9.8
As time of ascent = time of descent A
120
:. Total time taken = 5 + 5 = 10 s.
t 100
(ii) When the lift moves up with uniform speed : The
:§:
O
80
uniform speed of the lift does not change the relative
velocity of the ball w.r.t. the boy i.e. it still remains 49 ms -1. .r I
60
2 4 6 8 10
The belt moves with a speed of 4 kmh - 1. For an observer on a t(s) ~
stationary platform outside, what is the
(i) speed of the child running in the direction of motion Fig. 3.95
of the belt,
. 1
(ii) speed of the child running opposite to the direction of Ans. x (t ) = x (0) + v (0) t + - gt 2
motion of the belt, and 2
(iii) time taken by the child in (i) and (ii) ? If we take origin for position measurement on the
Which of the answers alter if motion is viewed by one of the ground, then the positions of the two stones at any instant
parents? . t will be
Ans. (i) Speed of the child running in the direction of Xl = 200 + 15 t - t x 10 t 2
...(1)
= 200 + 30 t - t x 10 t
motion of the belt
x2 2 ...(2)
= (9 + 4) kmh-I =13 kmh-1•
MOTION IN A STRAIGHT LINE 3.53
When the first stone hits the ground, Similarly, acceleration of the particle during journey
xl = 0 AB is given by
v = u + at or 0 = 12 + a x 5
or 200 + 15 t - 5t 2 = 0
or a = - 2.4 ms - 2
or 5t 2 - 1St - 200 = 0
2 Velocity of the particle after 2 s from start will be
or t - 3t + 40 = 0
v = u + at =0 + 2.4 x 2 = 4.8 ms - 1
3 ± ~9 + 160 3 ± 13
.. t = =--=8s or -5s :. Distance covered by the particle between t = 2 to 5 s
2 2 (in 3 s) is given by
As time cannot be negative, so t =8s
~ = ut + t at2
R
3+t
i.e. the first stone hits the ground after 8 s. 2 = 25.2 m
= 4.8 x x 2.4 x 3
From (1) and (2), the relative position of second stone
w.r.t. first is given by Distance covered by the particle in t = 5 to 6 s (in 1 s) is
given by
x2 - xl = 15 t
As there is a linear relationship between x2 - xl and t, S2 = ui + t at 2
SI
so the graph is straight line OA upto t = 8 s. After t = 8 s, = 12 x 1+ t x (- 2.4) x 12 = 1O.S m
only the second stone is in motion. So the graph is
parabolic (AB) in accordance with quadratic equation, Total distance travelled in t = 2 to 6 s,
After t = 10 s, the separation between the balls is zero, 3.28. The velocity-time graph of particle in one-dimensional
IT
which explains the part BC of the graph. motion is shown in Fig. 3.97.
3.27. The speed-time graph of a particle moving along a
fixed direction is shown in Fig. 3.96. Obtain the distance travelled
by the particle between (j) t = 0 to 10 s (ii) t = 2 to 6 s. What is
the average speed of the particle in intervals in (j) and (ii) ?
H
Fig. 3.97 0 t1 t2 t~
Which of the following formulae are correct for describing
the motion of the particle over the time interval tl to t2 :
O
1. Are rest and motion absolute or relative terms? 19. If the instantaneous velocity of a particle is zero,
2. Can an object be at rest as well as in motion at the will its instantaneous acceleration be necessarily
same time? zero?
R
3. Is it true that a body is at rest in a frame within 20. A woman standing on the edge of a cliff throws a
which it has been fixed ? ball straight up with a speed of 8 kmh -1 and then
throws another ball straight down with a speed
4. Under what condition can an object in motion be
of 8 kmh -1 from the same position. What is the
considered a point object?
ratio of the speeds with which the balls hit the
5. Give an example of a physical phenomenon in ground?
SI
which earth cannot be regarded as a point mass.
21. A body travels, with uniform acceleration ~ for
6. Under what condition will the distance and time t1 and with uniform acceleration a2 for time t2.
displacement of moving object have the same What is the average acceleration?
magnitude? [Chandigarh 08)
22. What is the nature of position-time graph for a
7. A bullet fired vertically upwards falls at the same uniform motion? [Chandigarh 031
place after some time. What is the displacement of
23. What does the slope of position-time graph
the bullet?
indicate? [Himachal 07)
8. A particle is moving along a circular track of radius r.
IT
24. What is the nature of velocity-time graph for
What is the distance traversed by particle in half
uniform motion?
revolution? What is its displacement?
25. If the displacement-time graph for a particle is
9. Will the displacement of an object change on
parallel to displacement axis, what should be the
shifting the position of origin of the coordinate
velocity of the particle?
system ? [Himachal 06C)
26. If the displacement-time graph for a particle is
10. What does the speedometer of a car measure-
H
parallel to time-axis, how much is the velocity of
average speed or instantaneous speed ?
the particle?
11. What is the numerical ratio of velocity to speed of
27. How can the distance travelled be calculated from
an object?
the velocity-timegraph in a uniform one-dimensional
12. A ball hits a wall with a velocity of 30 ms -1 and motion?
O
object.
the case of a body projected vertically upwards
15. Give an example which shows that a negative from the ground after reaching the ground ?
accele ration can be associated with a speeding up
30. Can a particle with zero acceleration speed up ?
object.
16.. Is the acceleration of a car greater than when the
-t
31. Is the formula: s = vt - at2 correct, when the
body is moving with uniform acceleration?
accelerator is pushed to the floor or when brake
pedal is pushed hard ? 32. A body projected up reaches a point Pof its path at
the end of 4 seconds and the highest point at the
17. The v-t graphs of two objects make angles of 30 0
18. Is it possible that your cycle has a northward 33. Can a body subjected to a uniform acceleration
velocity but southward acceleration? If yes, how? always move in a straight line ?
MOTION IN A STRAIGHT LINE 3.55
34. Suggest a suitable physical situation for the graph 40. What does the slope of velocity-time graph
shown in Fig. 3.89(b) on page 3.50. . represent? [Himachal07; Delhi 10]
35. A uniformly moving cricket ball is turned back 41. What does the area under velocity-time graph
by hitting it with a bat for a very short time- represent? [Himachal07]
interval. Suggest acceleration-time graph for the 42. What does the area under acceleration-time graph
situation. represent?
36. The position coordinate of a moving particle is given 43. The displacement-time graphs for the two particles
by x = 6 + 1St + 9t2 (x in metres and t in seconds). A and B are straight lines inclined at angles of 30°
What is its velocity at t = 2 sec. ? [Chandigarh03] and 45° with the time-axis. What is the ratio of the
37. A player throws a ball upwards with an initial velocities v A : VB ? [Delhi 12]
R
speed of 29.4 m 5-1. What are the velocity and 44. Is the time variation of position, shown in the figure
acceleration of the ball at the highest point of its below, observed in nature? [CentralSchools121
motion ? [Delhi05; Central Schools12]
38. Under what condition will the distance and
displacement of a moving object have the same
SI
magnitude ? [Ch~ndigarh08]
39. State the condition when the magnitude of velocity
and speed of an object are equal. [Delhi 081
Answers
1. Yes, both rest and motion are relative terms. 14. An object with positive acceleration is slowing
down if its initial velocity is negative.
IT
2. Yes.A body may be at rest relative to one object and
at the same time it may be in motion relative to 15. An object in simple harmonic motion speeds up
another object. while moving from an extreme position to the mean
3. Yes. position but its acceleration is negative.
4. An object can be considered a point object if its size 16. Acceleration is greater in the second case, because
is much smaller than the distance travelled by it. car suddenly comes to halt, the rate of change of
5. Solar or Lunar eclipse. velocity is large.
H
!i = Slope of v-t graph of first object
6. When the object moves along a straight line in the 17.
same fixed direction. a2 Slope of v-t graph of second object
7. Zero. tan 30° 1/ J3 1
= -- = -- = - =1: 3.
8. Distance travelled = 1t r. tan 60° J3 3
O
23. The slope of position-time graph gives velocity of 35. The given velocity-time graph shown in Fig. 3.89(b)
the object. represents the motion of a ball thrown up with
24. For uniform motion, velocity-time graph is a straight some initial velocity and rebounding from the floor
line parallel to time-axis. with reduced speed after each hit.
R
37. At the highest point, the velocity of the ball is zero
28. Area under acceleration time graph for any time and its acceleration is equal to acceleration due to
interval gravity acting in the downward direction.
= Change of velocity of the body in that interval. 38. When the body moves along a straight line path.
29. Zero. 39. When the body moves along a straight line path.
SI
30. Not possible. 40. Acceleration.
31. Yes. 41. Displacement.
42. Change in velocity in the given time interval.
s = (U+V)
-2- xt = (v-at+v)
2 xt = vt -"21 at 2 .
43. vA = tan 30° = 1/.J3 = 1 : .J3
32. 12 + (12 - 4) = 20 seconds. VB tan 45° 1
33. No. The path of a projectile is a parabola even when 44. No, a body cannot occupy two different positions at
it has a uniform acceleration. the same instant of time.
IT
34. Refer to the answer of Exercise 3.19(b) on page 3.50.
2 or 3 Marks Each
H
1. Define the following terms and write the SI units: 8. Using integration technique, prove that
(a) Displacement, and v2 - if = Zas. [Himachal04,09]
(b) Instantaneous velocity. [Delhi 03C] 9. Prove that
2. Distinguish between average velocity and instan-
taneous velocify. If the velocity does not change
x = Xo + vot + !at 2 [Delhi 1998; Himachal 03,04]
O
14. Derive the three kinematic equations for 19. Acceleration-time graph of a moving object is
uniformly accelerated motion graphically. shown in figure. Draw the velocity-time graph and
[Delhi 06] displacement-time graph corresponding to this
15. Derive the relation graphically: type of motion.
s=ut+1at2
2 t
§
where symbols have their usual meanings. :.:
e ~------------
1
[Himachal 07 ; Chandigarh 07]
R
Time-+
where symbols have their usual meanings.
[Himachal 07] Fig. 3.98
17. Explain with example the distinction between the 20. Draw the following graphs (expected nature only)
magnitude of average velocity over an interval of between distance and time of an object in case of
SI
time and the average speed over the same period. (i) For a body at rest
Show that the average speed is either greater than
(ii) For a body moving with uniform velocity
or equal to the average velocity. When is the
equality sign true ? [Delhi 03C] (iii) For a body moving with constant acceleration.
[Central Schools 09]
18. Draw the following graphs for an object projected
21. Draw the following graphs (expected nature only)
upward with a velocity vO' which comes back to the
representing motion of an object under free fall.
same point after some time:
Neglect air resistance.
(i) Acceleration versus time graph.
(i) Variation of position with respect to time.
IT
(ii) Speed versus time graph.
(ii) Variation of velocity with respect to time.
(iii) Velocity versus time graph.
[Central Schools 03 ; Delhi 12]
(iii) Variation of acceleration with respect to time.
[Central Schools OS]
H
Answers
1. Refer to points 13 and 23 of Glimpses. 13. Refer answer to Q. 28 on page 3.23.
2. For average velocity and instantaneous velocity, 14. Refer answer to Q. 29 on page 3.24.
O
refer answer to Q. 11 on page 3.5. No. The average 15. Refer answer to Q. 29 on page 3.24.
velocity will be same for different time intervals if
16. Refer answer to Q. 29 on page 3.24.
the instantaneous velocity does not change from
instant to instant. 17. Refer answer to Exercise 3.13 on page 3.48.
3. Refer to points 17 and 18 of Glimpses. 18. (a) See Fig. 3.52 on page 3.27.
(b) See Fig. 3.50 on page 3.27.
M
6. Refer answer to Q. 21(i) and (ii) on page 3.11. 19. The object is moving with a constant acceleration.
(i) For velocity-time graph, see Fig. 3.43 on
7. Refer answer to Q. 21 (iii) on page 3.11.
• page 3.26.
8. Refer answer to Q. 21(iii) on page 3.11.
(ii) For displacement-time graph, see Fig. 3.38 on
9. Refer answer to Q. 20(ii) on page 3.10. page 3.25.
10. Refer answer to Q. 21(iv) on page 3.12. 20. (i) See Fig. 3.29 on page 3.25
11. Refer answer to Q. 24 on page 3.22. (ii) See Fig. 3.30 on page 3.25.
12. Refer answer to Q. 26 on page 3.23. (ijj) See Fig. 3.31 on page 3.25.
3.58 PHYSICS-XI
r
t
§
:8
~t-__a_=""g __
~
~ ~
<
Time ....) Time....)
(i) (ii) (iii)
Fig. 3.99
R
5 Marks Each
SI
1. (a) With the help of a simple case of an object 3. Draw velocity-time graph of uniformly accelerated
moving with a constant velocity, show that motion in one dimension. From the velocity-time
the area under velocity-time curve graph of uniform accelerated motion, deduce the
represents the displacement over a given equations of motion in distance and time.
time interval. [Chandigarh 04]
(b) Establishthe relationx = vel + t at 2 graphically. 4. Define relative velocity of one object w.r.t. another
(c) A car moving with a speed of 126 Ian h -1 is object. Draw position-time graphs for two objects
brought to a stop within a distance of 200 m. moving along a straight line ; when their relative
Calculate the retardation of the car and the velocity is (i) zero and (ii) positive.
IT
time required to stop it. [Delhi 03] [Himachal06 ; Central Schools 12]
2. Derive an equation for the distance covered by a
uniformly accelerated body in nth second of its
motion. A body travels half its total path in the last
second of its fall from rest, calculate the time of its
fall. [CentralSchools05]
H
Answers
or t=2±.fi
h=0+'!gt2 ...(/) or t = 3.41 s or 0.59 s (neglected)
2
h 1 3. Refer answer to Q. 29 on page 3.24.
and - = 0 + - g (2t -1) ...(ii)
2 2 4. Refer answer to Q.35 on page 3.33. See Fig. 3.72 and
Fig. 3.73 on page 3.34.